Download as pdf or txt
Download as pdf or txt
You are on page 1of 88

Previous Year Question

nta net GeoGraPhY


No Subject Page No
1 NTA NET Geography November 2020………………………………. 1
2 Nta NET Geography December 2021 (Shift-1)……………………… 15
3 Nta NET Geography December 2021 (Shift-2)……………………… 28
4 Nta NET Geography July 2022………………………………………. 41
5 NTA NET Geography February 2023………………………………... 55
6 NTA NET Geography June 2023…………………………………….. 68
PYQ 2020 BSSEI (M-8777279548) Coaching for NET Geography

NTA NET Geography November 2020


1. The slope replacement was propounded by
A) A. Strahler B) A. Penck
B) C) W.M. Davis D) H. Wood

2. Which one of the following methods commonly used to estimate numerically the change taking place in
one sector or industry bringing about changes in other sectors of the economy?
A) Cluster analysis B) Econometrics
C) Input – Output analysis D) Principal component analysis

3. Given below are two statements: one is labeled as Assertion (A) and the other is labeled as reason (R):
Assertion (A): Thermal infrared energy in remote sensing is sensed by radiometers.
Reasons (R): Thermal infrared can neither be seen by human eyes nor can be photographed.
In the light of the above statements, choose the most appropriate answer from the options given below :

(A) Both (A) and (R) are correct and (R) is the correct explanation of (A)
(B) Both (A) and (R) are correct but (R) is NOT the correct explanation of (A)
(C) (A) is correct but (R) is not correct
(D) (A) is not correct but (R) is correct

4) Which one of the following latitudes has ‘no surplus’ and ‘no deficit’ heat balance in northern
hemisphere?
A) 180 B) 231/20 3) 38 0 4) 661/20

5) Identify the correct Relative Humidity (%) of an air mass with saturation mixing ratio at 250 C is 20
grams and H2O vapour content is 5 grams
A) 15% B) 30% 3) 25% 4) 50%

6) Given below are two statements : One is labeled as Assertion (A) and the other is labeled as Reason (R) :

Assertion (A): The globalization of production is concomitant of the globalization of trade. As one can’t
function without the other.
Reasons (R) : The scale, volume and effective of international trade have all continued to increase since the
1970s due to ongoing process of global space / time convergence.
In the light of the above statements , choose the most appropriate answer from the options given below :
A) Both (A) and (R) are correct and (R) is the correct explanation of (A)
B) Both (A) and (R) are correct are but (R) is not the correct explanation of (A)
C) (A) is correct but (R) is not correct
D) (A) is not correct but (R) is correct

7) Rimland theory was presented by M.J. Spykman in


A) 1941 B) 1942 C) 1964 D) 1944

8) Who among the following scholars advocated a policy of deliberate imbalanced development ?
A) G. Myrdal B) A.O Hirchman C) A.G Frank D) R.Nurke

9) Which one of the following curves is correct to show the cumulative frequency distribution
A) Frequency curve B) Ogive
C) Long normal curve D) Semi-log curve

10) Two dimensional diagrams are also known as


A) Size B) Volume C) Ratio D) Area

1
PYQ 2020 BSSEI (M-8777279548) Coaching for NET Geography
11) Given below are two statements : One is labeled as Assertion (A) and the other is labeled as Reason (R)
Assertion (A): Globalization raises the degree of independence resulting in lessor control over the domestic
economy and loss of economy sovereignty.
Reasons (R) : Globalization promotes consumption cultures that may not be in consonance with the lifestyle
and values of all countries.
In the light of the above statements, choose the most appropriate answer from the options given below:
A) Both (A) and (R) are correct and (R) is the correct explanation of (A)
B) Both (A) and (R) are correct but (R) is not the correct explanation of (A)
C) (A) is correct but (R) is not correct
D) (A) is not correct but (R) is correct

12) Match List I with List II:


List I-(Name of Author) List II-(Name of Book)

(A) A. Weber (1) Industrial Location:


An Economic Geographical
Analysis.
(B) E.M Hoover (2) Behavior and
Location: Foundations
for a Geographic and
Dynamic location theory.
(C) A. Pred (3) The location of
economic activity.
(D) D.M. Smith (4) Theory of the
location of the industries
(A) (A) 1 (B) 2 (C) 3 (D)4
(B) (A) 4 (B)2 (C)1 (D)3
(C) (A) 3 (B)1 (C)2 (D)4
(D) (A) 4 (B) 3 (C) 2 (D)1

13) Digitizing or scanning are example of which GIS procedure:


A) Data Output B) Data acquisition
C) Edge matching D) Error detection

14) ‘Dry Point’ settlement are common in


A) Deltas B) Mountain regions
C) Deserts D) Rain shadow areas

15) International boundary between India and Pakistan was demarcated after the partition. Which type of
boundary is this

A) Cultural B) Complex C) Antecedent D) Consequent

16) Which of the following statements are true for vector data models in GIS.
(A) It is a simple data model and has low cartographic output quality
(B) It is a simple data structure and has high cartographic output quality
(C) It is characterized by case of data processing
(D) It is cheap data collection process as it was less expensive technology

Choose the appropriate answer from the options given below:


A) (A) only B) (B) only
B) C) (C) only D) (D) only

2
PYQ 2020 BSSEI (M-8777279548) Coaching for NET Geography
17) Given below are two statements : One is labeled as Assertion (A) and the other is labeled as Reason (R)
Assertion (A): The land capability approach is that where land was graded according to its suspectibility
to soil erosion under differing farming and forestry systems of production.

Reasons (R): The most demanding system of production would therefore concentrate on the best
and most extensively on the poorest land.

In the light of the above statements, choose the most appropriate answer from
the options given below :
A) Both (A) and (R) are correct and (R) is the correct explanation of (A)
B) Both (A) and (R) are correct but (R) is not the correct explanation of (A)
C) (A) is correct but (R) is not correct
D) (A) is not correct but (R) is correct

18) Which one of the following elevations upto that Koppen’s Af type of climate extends from the mean sea
level?

A) 1000m B) 1500m C) 2000m 4) 2500m

19) Who according to Hartshorne had set the direction of geographic thought for the future?

A) Huntington E.W. B) Videl de la Blacho


B) E) E Semple D) Van Richthofen

20) Which of the following is not associated with Political Geography


A) I. Bowman B) R. Johnston C) F. Ratzel D) H.J Mackinder

Choose the correct answer from the options given below :


A) (A) Only B) (A) and (B) only
C) (B) only D) (B) and (C) only

21) Given below are two statements : One is labeled as Assertion (A) and the other is labeled as Reason (R)
Assertion (A): Laterite soil is formed under the conditions of high temperature and high rainfall with
alternate wet and dry periods.

Reasons (R): The laterite soils swell greatly and become stricky when wet in rainy season and also known as
Self ploughed

In the light of the above statements, choose the most appropriate answer from the options given below:

A) Both (A) and (R) are correct and (R) is the correct explanation of (A)
B) Both (A) and (R) are correct but (R) is NOT the correct explanation of (A)
C) (A) is correct but (R) is not correct
D) (A) is not correct but (R) is correct

22) Given below are two statements :


Statement I: The consumption of oxygen has increased after the industrial revolution in 1860.

Statement II. The Brazil and India uses more oxygen annually than other countries of the world.

In the light of the above statements, choose the correct answer from the options given below:
A) Both statement I and Statement II are true
B) Both statement I and statement II are false
C) Statement I is correct but statement II is false
D) Statement I is incorrect but statement II is true

3
PYQ 2020 BSSEI (M-8777279548) Coaching for NET Geography
23) Concept of ‘Primate City” was given by Jefferson in
A) 1929 B) 1938 C) 1939 D) 1940

24) ‘Household register’ in Japan was known by the name


A) Takeshi B) Yumitava C) Koseki D) Dketana

25) Given below are two statements :


Statement I: All moving objects, by virtue of their motion are of doing work.
Statement II: Potential energy is the energy of motion.
In the light of the above statement , choose the correct answer from the options given below:
A) Both statement I and statement II are true
B) Both statement I and statement II are false
C) Statement I is correct but statement II is false
D) Statement I is incorrect but statement II is true

26) Which one of the following is the correct sequence of contributors in the development of Greek
geography?
A) Anaxiamander, Hipparchus , Herodotus, Eratosthenes
B) Eratosthenes, Herodotus , Hipparchus, Anaximander
C) Anaximander, Herodotus, Eratosthenes, Hipparchus
D) Hipparchus, Anaximander , Herodotus , Eratosthenes

27) (Type of Clouds)


(A) Altostratus (B) Stratocumulus
(C) Stratus (D) Cirrus

Choose the correct answer from the options given below : (upward sequence)
A) (A) , (B), (D),(C) B) (C),(B),(A),(D)
C) (B),(C),(A),(D) D) (C),(B),(D),(A)

28) Which of the following two sources carbon is released?


(1) Volcanic eruption (2) Combustion of fossil fuels
(3) Landslide (4) Open mining

Choose the correct answer from the options given below:

A) (1) and (2) only B)(2) and (3) only


C)(1) and (3) only D) (2) and (4) only

29) The ‘Arabian sea Branch’ and Bay of Bengal Branch of Indian monsoon merge together over which of
the following region.
A) The Punjab and adjoining Himalayas B) Central India
C) North East Indian States D) Western Ghats

30)Match List I with List II:

List I List II
(A)Heartland (1)Van Valkerburg
(B)Rimland (2)Ratzel
(C)Classification of state (3)Spykman
(D)Law of territorial growth (4)Mackinder
Of states
Choose the correct answer from the options given below:
A) (A)-(4), (B)-(3), (C)-(1), (D)-(2) B) (A)-(3), (B)-(4), (C)-(2), (D)-(1)
C) (A)-(1), (B)-(2), (C)-(3), (D)-(4) D) (A)-(2), (B)-(1), (C)-(4), (D)-(3)

4
PYQ 2020 BSSEI (M-8777279548) Coaching for NET Geography
31) Which one of the following techniques is correct to show inequality of two given distributions.
A) Rn statistics B) X2 t test
C) Correlation Coefficient D) Lorenz curve

32) Ria is an example of :


A) Submerged upland shore B) Emerged upland shore
C) Neutral Shore D) Compound Shore

33) Match List I with List II:

List I- Theories List II- Proponents


(A)Thermal Contraction Theory (1) James Hutton
(B) Theory of Geosyncline (2)Jeffreys. J
(C) Theory of Seafloor spreading (3)Hall and Dana
(D) Doctrine of Uniformatarianism (4)Harry Hess
Choose the correct answer from the options given below:
A)(A)-(1), (B)-(2), (C)-(3), (D)-(4)
B) (A)-(4), (B)-(3), (C)-(1), (D)-(2)
C) (A)-(3), (B)-(2),(C)-(1), (D)-(4)
D) (A)-(2), (B)-(3), (C)-(4), (D)-(1)

34)Read the following statement regarding Hypsometric curve


(A)It shows the area- height relationship of a drainage basin.
(B) A convex curve indicates that bulk of the basin’s area remains at relatively low elevation.
(C) A concave curve is characterized by diffusive hill slope processes such as land sliding, soil creep etc.
(D) The simple way to characterize the shape of hypsometric curve for any drainage basin is to calculate
hypsometric integral (HI).
Choose the correct answer from the options given below:
A) (A), (B) and (C) are correct
B) (B) and (C) are correct
C) (A) and (D) are correct
D) (A), (B), (C) and (D) are correct

35) Which one of the following type of landforms differs from the other on the basis of its mode of
A) Rapids B) Escarpments C) Quester D) Dripstones

36) Given below are two statements : One is labelled as Assertion (A) and the other is labelled as Reason
(R):
Assertion (A): Aleutian Island are a site of volcanic eruptions.
Reasons (R): Convergent plate margins are of volcanic eruptions.
In the light of the above statement, choose the most appropriate answer from the options given below:
A)Both (A) and (B) are correct and (R) is the correct explanation of (A)
B)Both(A)and(R)are correct but(R)is NOT the correct explanation of(A)
C) (A) is correct but (R) is not correct
D) (A) is not correct but (R) is correct

37) In that of the following maps average slope and relative relief are shown simultaneously
A) Trachographic map B) Planimetric map
C) Hypsometric map D) Morphological map

38) Which one of the following recycle periods is correct for oxygen ?
A) 100 years B) 2000 years C) 1000 years D) 500 years

5
PYQ 2020 BSSEI (M-8777279548) Coaching for NET Geography
39) The characteristics of the countries engaged in subsistence farming include
1) Developing economy 2) High density of population
3) Low level of technology 4) High developed international trade.
Choose the correct option:-
A) 1,2,4 B) 2,3,4 c) 2,3 D) 1,2,3

40) Who among the following defined geography as a Chorologicl science ?


A) P.E James B) Hettner.J C) Richthofen D) Darwin. C

41) Which one of the following percentage of frequencies between the range of 0 to +1.00 (𝜎) standard
deviation .

A)34.13% B) 38.23% C) 46.20% D) 47.73%

41) Match List I with List II


List I List II
A. Fundemental Ecology 1.H.K. Gupta
B. India’s water wealth 2. D.N. Wadia
C. Geology of India 3. E. P. Odum
D. Dams and Earthquakes 4. K.L. Rao
Codes:-
A B C D
(a) 3 2 4 1
(b) 3 4 2 1
© 2 3 1 4
(d) 4 1 2 3

42) Given below are two statements: One is labeled as Assertion (A) and the other is labeled as Reason (R) :

Assertion (A): In areas of intensive subsistence agriculture, animal husbandry is a meager sideline of crop
farming.
Reasons (R): Through out the region , the farmers are handicapped in raising cattle due to lack of scientific
methods of breeding the animals.
In the light of the above statements, choose the most appropriate answer from the options given below :
A)Both (A) and (R) are true and (R) is the correct explanation of (A)
B) Both (A) and (R) are true but (R) is not the correct explanation of (A)
C) (A) is true but (R) is false
D) (A) is false but (R) is true

43) Which one of the following components does not match with the Darwinian theory of Origin of species
?
A)Natural hazard B) Natural selection
C) Adaptation D) Survival of the fittest

6
PYQ 2020 BSSEI (M-8777279548) Coaching for NET Geography
44) Histogram is associated with which one of the following data distribution
A) Discreate frequency distribution B)Continuous frequency distribution
C) Log – normal frequency distribution D)Un- classified data

45) Which one of the following rivers does not flow through Himachal Pradesh
A) Beas B) Chenab C) Jhelum D) Ravi

46) Which among the following is an example of a ‘nodal region’


A) Damodar valley region B) National capital region
C) Western Ghat region D) Upper ganga – Yamuna doab region

47) An introduction to the ‘application of geography’ and ‘The Geographical distribution of mankind’ are
the work of:
A) Card Ritter B) Friedrich Ratzel
C)Vidal de la Blache D) Alfred Hettner

48) Given below are two statements:


Statement I: In any system of constant mass, energy is neither created nor destroyed.
Statement II: The energy flow in the ecosystem is NOT unidirectional.
In the light of the above statements, choose the most appropriate answer from the options given below:
A) Both statement I and statement II are correct
B) Both statement I and II are incorrect
C) Statement I is correct but statement II is incorrect
D) Statement I is incorrect but statement II is correct

49) Given below are two statements : One is labeled as Assertion (A) and the other is labeled as Reason (R)
Assertion (A): The locational – climatic factors put Mackinder’s Heartland at a relative disadvantage as
compared to certain other larger and well to do areas of the world.
Resons (R) : Heartland was a region of permanent difficulties because of its locations and resultant extreme
climate.
In the light of the above statements, choose the most appropriate from the options given below :
A)Both (A) and (R) are correct and (R) is the correct explanation of (A)
B)Both(A) and(R)are correct but(R)is NOT the correct explanation of(A)
C) (A) is correct but (R) is not correct
D) (A) is not correct but (R) is correct

50) Which of the following earthquake waves are more destructive at the epicenter?
A) S- Waves B) P- Waves C) R- Waves D) L- Waves

51) List I (Type of precipitation ) List II I(Approximate size)


(A) Mist (1) 2-5 mm
(B) Snow (2) 0.5-5 mm
(C) Graupel (3) 0.005-0.05 mm
(D) sleet (4) 1 mm – 20 mm

Choose the correct answer from the options given below :


A) (A)-(1), (B)-(3), (C)-(2), (D)-(4) C) (A)-(4), (B)-(2), (C)-(3), (D)-(1)
B) (A)-(3), (B)-(4), (C)-(2), (D)-(1) D) (A)-(3), (B)-(4), (C)-(1), (D)-(2)

52) Study in social and health issues , domestic politics and security issues and international diplomacy is
new trend in political geography . it is termed as
A) Neo-politics B) Meta – Geopolitics
C) Mega – politics D)Developmental-Geopolitics

7
PYQ 2020 BSSEI (M-8777279548) Coaching for NET Geography
53) The Arab scholar who made corrections to ptolemy’s book was
A) Ibn – Batuta B) Ibn- Khaldun C) AI-Masudi D) AI- Idrisi

54) Which one of the following processes is related to absorption of inorganic salts and water by plant roots
?
A) Oxidation B) Osmosis C)Hydration D) Hydrolysis

55) What is not true about international trade in post – globalization phase:
A) It has enhanced the spatial integration of world economy
B) Role Of digital technologies in facilitation of trade has become increasing important.
C)Regional comparative advantage is on decline.
D) The ability of a country to compete in international trade and economy is highly dependent on the
transport system as well trade facilitation frame work.

56) Which of these is not a type of federal state?


A) Compromised B) Imposed C) Composite D) Centralised

57) Match list I with II

List I- Land Froms List II- Agents of Denudations


(A)Monadnocks (1) Wind
(B) Blind Valley (2) Glacier
(C) Drumlin (3) River
(D) Dreikantes (4) Groundwater
Choose the correrct answer from the options given below:
A)(A)-(3), (B)-(4), (c)-(2), (D)-(1)
B) (A)-(2), (B)-(3), (C)-(4), (D)-(1)
C) (A)-(1), (B)-(4), (C)-(3), (D)-(2)
D) (A)-(4), (B)-(2), (C)-(1), (D)-(3)

58) Which one of the following is not a non renewable resources?


A) Fish B) Plants C) Fossil fuels D) Solar energy

59) Given below are two statements : One is labeled as Assertion (A) and the other is labeled as Reason (R)
Assertion (A): Build operate transfer (BOT) is a scheme under which public operators are invited to
construct roads and bridges.

Reasons (R) : The national highways act has been amended to facilitate private investment under
(BOT) scheme.
In the light of the above statements , choose that most appropriate answer from the options given below:
A)Both (A) and (R) are correct and(R) is the correct explanation of (A)
B)Both(A) and(R) are correct but(R)is NOT the correct explanation of(A)
C) (A) is correct but (R) is not correct
D) (A) is not correct but (R) is correct

60) Which among the following statement are true for deceleration in agricultural growth in post reform
period.
(A)Deceleration in public and general investment in agriculture.
(B) Dwindling farm size
(C) Failure to develop new technologies
(D)Progressive land reforms.
Choose the correct answer from the options given below:
A) (A) , (B), (C) and (D) B) (A) and (B) only
C) (B) and (D) only D) (A), (B), and (C) only

8
PYQ 2020 BSSEI (M-8777279548) Coaching for NET Geography
61) Given below are two statements : One is labeled as Assertion (A) and the other is labeled as Reason (R)
Assertion (A): The moon’s high tide time lag in each is 50 minutes.
Reasons (R): The total duration required to complete the two cycles of high and low at any place is 24 hour
50 minutes.
In the light of the above statements, choose the most appropriate answer from the options given below:
A)Both (A) and (R) are correct and (R) is the correct explanation of (A)
B) Both(A) and(B) are correct but(R) is NOT the correct explanation of(A)
C) (A) is correct but (r) is not correct D) (A) is not correct but (R) is correct

62. Consider the following statements regarding the Weaver’s method of crop combination.
(A)He proposed a method of analysis superior to simple inspection of the relative weights of the enterprises
in qa given area.
(B) He added a time dimension to the notion that Britain’s best and worst agriculture land has not changed
during last century.
(C) He sought to find the number of enterprises that minimised the difference between the actual and
theoretical enterprise combinations.
(D) He adopted a measure of yield defined as the potential production of one acre of good average farmland
under good management
Choose the correct answer from the options given below:
A)(A), (B) and (C) only B) (A), (C) and (D) only
C) (A) and (B) only D) (A) and (C) only

63) Which of the following groups of landforms is produced by erosion?


A) Playa, Pinnacles , Pot Holes B) Arate , Kame , Levees
C) Dramlin , Fijord, Wave built Platform D) Drip Stone , Sinkholes, U-Shaped Valley

64) Who enunciated the four stags in the evolution of space economy?
(A)John Friedmann (B) Karl Marx C) Ronald F.Abler D) August Losch

Choose the correct answer from the options given below:


A)(A) only B) (B) only C)(C) only D) (D) only

65) Which one of the following surface areas covered by a solar beam of 1 m width at noon with vertical
sunshine?
A) 1 m Width B) 4 m Width C) 45m Width D) 90 m Width

66. Identify the person who defined pollution as


“The presence at large of substances, or energy patterns which have been involuntarily produced, have
outlived their purpose, have escaped by accident, or have unforeseen effects, in quantities which harm his
(man’s) health or do offended him”
A)Lord Kenett B) D.M Dixon C) R.F. Dasmann D) A. Goudie

67. Who is recognized as the founder of scientific Marine Meteorology ?


A) L.C. King B) M.F. Maury C) W.M. Davis D) F.P. Shepard

68. Given below are two statements : One is labeled as Assertion (A) and the other is labeled as Reason (R) :
Assertion (A): At present the government sector is the largest consumer of information technology (IT)
services in India.
Reasons (R) : The Indian companies have out performed MNC’s in the total revenue and even IT export
share.
In the light of the above statements, choose the most appropriate answer from the options given below:
A)Both (A) and (R) are correct and (R) is the correct explanation of (A)
B)Both (A) and ® are correct but (R) is not the correct explanation of (A)
C) (A) is correct but (R) is not correct
D) (A) is not correct but (R) is correct.

9
PYQ 2020 BSSEI (M-8777279548) Coaching for NET Geography

69. Which one of the following connectivity matrices is correct to show the given network of six nodes ?

Answer- Option 2 or B.

70. There are three well – defined phases in International trade. The second phase, which is market with
mobility of factors of production, remainded in focus during
A) 1950s and 1960s B)1960s and 1970s
C) 1970s-1990s D) 1990s onward

71) Match List I with List II


List I(Book Name) List II(Authors)
(A)Fundamentals of Ecology (1)H.K.Gupta
(B)India’s Water Wealth (2)D.N. Wadia
(C)Geology of India (3) E.P. Odum
(D)Dams and Earthquakes (4)K.L. Rao
Choose the correct answer from the options given below:
A)(A)-(3), (B)-(2), C-(4), (D)-(1)
B) A)-(3), (B)-(4), (C)-(2), (D)-(1)
C) (A)-(2), (B)-(3), (C)-(1), (D)-(4)
D) (A)-(4), (B)-(1), (C)-(2), (D)-(3)

72. “Hyper markets” are shopping centres located


A) Near the CBD B) Near rail- road junction
C) Out of town D) Near hospitals

73. “Zone of assimilation” and ‘Zone of discard’ terms are related of the delimitation of
A) Shape of a city B) Function of city
C) CBD of city D) Migration trends of a city.

74. Match List I with List II


List I- Mountain Passes List II-State
(A)The Banihal Pass (1) Uttarkhand
(B) The Bara Lacha La Pass (2)Sikkim
(C) The Jelap La Pass (3) Jammu and Kashmir
(D) The Lipu Lekh Pass (4) Himachal Pradesh
Choose the correct answer from the options given below:
A)(A)-(1), (B)-(4), (C)-(3), (D)-(2) B) (A)-(2), (B)-(1), (C)-(4), (D)-(3)
C) (A)-(3), (B)-(2), (C)-(4), (D)-(1) D) (A)-(3), (B)- (4), (C)-(2), (D)-(1)

10
PYQ 2020 BSSEI (M-8777279548) Coaching for NET Geography
75. Which one of the following codes correctly show the optimal location of profitability in different spatial
cost/ revenue situations in diagram below

(A) A (B) B (C) C (D)

76. Identify the processes of heating of ocean water


A) Absorption of radiation from the sun B) Transformation of kinetic energy
C) Melting of polar ice D) Violation of Ocean laws
Choose the correct answer from the options below:
A)(A) and (C) only B) (B) and (C) only
C) (A) and (B) only D) (C) and (D) only

77. Match List I with List II:


List I- Deposits List II-Agents
(A)Moraines (1) Wind
(B) Silt (2)Glacier
(C) Loess (3) Underground water
(D)Drip stones (4) River

Choose the correct answer from the options given below:


A)(A)-(2), (B)-(4), (C)-(1), (D)-(3) B) (A)-(1), (B)-(3), (C)-(2), (D)-(4)
C) (A)-(3), (B)-(2), (C)-(4), (D)-(1) D) (A)-(4), (B)-(1), (C)-(3), (D)-(2)

78. Given below are two statements : One is labeled as Assertion (A) and the other is labeled as Reason (R) :
Assertive(A): Globalization has resulted in widening inequalities between the forward and backward state in
India
Reasons (R): The organized sector provided 63% of net domestic product but provided employment to 93%
of work force.
In the light of the above statements, choose the most appropriate answer from the options given below:
A)Both (A) and (R) are correct and (R) is the explanation of (A)
B) Both(A) and(R) are correct but (R)is NOT the correct explanation of(A)
C) (A) is correct but (R) is not correct D) (A) is not correct but (R) is correct

79. Match List I With List II:


List – I(Measurment Methods) List II-(Devised by)
(A) Rank coefficient Method (1) Jasbir Singh
(B) Crop yield and concentration indices (2) Dudley stamp
(C) Carrying capacity of land (3) Kendall
(D) Index of productivity (4)M. Shafi

Choose the correct answer from the options given below:


A)(A)-(3), (B)-(1), (C)-(2), (D)-(4) B) (A)-(4), (B)-(3), (C)-(1), (D)-(2)
C)(A)-(1), (B)-(4), (C)-(2), (D)-(3) D) (A)-(2), (B)-(1), (C)-(4), (D)-(3)

11
PYQ 2020 BSSEI (M-8777279548) Coaching for NET Geography
80. Given below are two statements : One is labeled as Assertion (A) and the other is labeled as Reason (R) :
Assertion(A): Along with water vapour, carbon dioxide is largely responsible for the green house effect of
the atmosphere.
Reasons(R): Carbon dioxide is not an important heat absorber.
In the light of the above statements, choose the most appropriate answer from the options given below:
A)Both (A)and(R) are true and (R) is the correct explanation of (A)
B) Both(A) and (R)are true but (R) is not the correct explanation of (A)
C) (A)is true but (R) is false
D) (A) is false but (R) is true

81.Which of the following statements is/are not true for green revolution in India?
(A) Green Revolution ensured that there was no regional disparity
(B) Green revolution ensured increased productivity among selected crops.
(C) Green Revolution ensured uniform earnings among all farmers.
(D) Green Revolution ensured mechanization in farms.
Choose the correct answer from the options given below:
A)A,B,C and D B) Only A C) Only B D) Only D

82. Match List I With II:

Codes:-
A B C D
(a) II I IV III
(b) I II IV III
(c) II I III IV
(d) II IV I III

83.Sand dunes formed as long elongated rides oriented at right angles to the wind direction are knows as:
A) Star duns B) Barchans C) Transverse dunes D)Seifs

84. The rigid waves surrounding the Geosynclines were called by Kober as
A) Kratogen B) Orogen C) Primarumph D) Endrumph

85. Given below are two statements:


Statements I : The Darwinian theory of origin of species almost conform to the law of uniformitarianism.
Statement II: There are heritable variations in the individual species.
In the light of the above statements, choose the most appropriate answer from the options given below:
A)Both statement I and statement II are correct
B) Both statement I and statement II are incorrect
C) statement I is correct but statement II is incorrect
D) statement I is incorrect but Statement II is correct

12
PYQ 2020 BSSEI (M-8777279548) Coaching for NET Geography
86. Given below are two statements : One is labeled as Assertion (A) and the other is labeled as Reason (R) :
Assertion (A):Von Thunen argued that the sectoral fields are the most irrational, reducing the time spent by
the farmers in travelling around the farm./
Reasons (R): Sectoral fields are not common but they are used in varying farming systems and
environments.
In the light of the above statements, choose the most appropriate answer from, the options given below:
A)Both (A) and (R) are correct and(R) is the correct explanation Of (A)
B) Both (A)and(R) are correct but (R) is not the correct explanation of(A)
C) (A) is correct but (R) is not correct .
D) (A) is not correct but (R) is correct.

87. Given below are two statements:


Statement I : Due to an increase of pressure near the ocean bottom, the denisity also increases from surface
towards the depth.
Statement II: If light surface water is driven away by winds, dense sub-surface water upwells on the surface.
In the light of the above statements , choose the correct answer from the options given below:
A)Both statement I and statement II are true
B) Both statement I and statement II are false
C) Statement I is correct but statement II is false
D) Statement I is incorrect but statement II is true

88. Match List I with list II:

List I – Salts sea water List II- Amount in sea water(%)


(A)Sodium chloride (1) 3.807%
(B) Magnesium chloride (2) 27.213%
(C) Magnesium sulphate (3) 1.260%
(D)Calcium sulphate (4) 1.658%

Choose the correct answer from the options given below:


A)(A)-(2), (B)-(1), (C)-(4), (D)-(3)
B) (A)-(2), (B)-(1), (C)-(3), (D)-(4)
C) (A)-(1), (B)-(3), (C)-(4), (D)-(2)
D) (A)-(1), (B)-(2), (C)-(4), (D)-(3)

89. Which one of the following percentage of solar radiation reflected back and scattered to space?

A)58% B) 12% C) 30% D) 20%

90. Who among the following geographers laid down the foundation of dichotomy of General versus
special Geography ?
A) Immanuel Kant B) Sebstian Munster
C) Peter Apian D)Bernhard Varenium

Comprehension:
Instruction: Please read the following passage and answer the question 91-95
Tropical cyclones are closed low- pressure system , generally about 650km in diameter which bring violent
winds, torrential rainfall and thunderstorms. Generally, they contain a central region, the eye ,with a diameter
of about then kilometers with light winds and more or less lightly clouded sky. In order for cyclones to develop
various conditions need to be fulfilled. There must be a plentiful supply of moisture, and for this reason
distribution of cyclones is closely related to those regions where the highest sea- surface temperatures of at
least 270 C . This moisture provides the necessary latent heat to drive the storm and to provide the rainfall.
Tropical cyclones tend to occur mainly in between the Tropic of Capricorn and Tropic of cancer except the
equatorial region i.e.50 on both side of the equator because closer than that to the Equator the Coriolis

13
PYQ 2020 BSSEI (M-8777279548) Coaching for NET Geography
parameter approaches Zero. The main cyclone activity in the Northern Hemisphere is in late summer and
autumn during the time of the equatorial trough’s northern displacement.

91. Which of the following conditions is essential to develop a tropical cyclone ?


A)Low wind speed
B) Mixing of cold and warm ocean currents
C) Plentiful supply of moisture
D) Mixing of saline and sweet water

92. The cyclone activity in Northern Hemisphere is associated with


A)Spring and winter B) Spring and Autumn
C) Autumn and winter D) Summer and Autumn

93. Which one of the following sea-surface temperature is idea for the formation of tropical cyclone ?
A) 270 C B)200 C C) 170 C D) 150 C

94 Which of following is the regional name of tropical cyclone in West Indies ?


A)WillyWilly B) Hurricane C) Typhoon D) Cyclone

95. Which one of the following elements is not associated with tropical cyclones?
A) Violent Winds B) Torrential rainfall C) Thunderstorms D) Dense fog

96. Comprehension
Passage
Consider the following data(96-100) and answer questions:

S.NO Class limit Frequency


1 100-200 4
2 201-300 12
3 301-400 24
4 401-500 40
5 501-600 16
6 601-700 12
7 701-800 10
8 801-900 5
9 901-1000 2
96. Which one of the following is the arithmetic mean value for the given data set
A)125 B)40 C)483 D)603

97.Which one of the following is the relative frequency in percentage for class limit 301-400 from
A) 18.2 B)19.2 C)20.2 D)24.2

98. Which one of the following is the cumulative frequency for the class limit 401-500 from the given
data set.
A) 24 B) 40 C) 16) D) 80

99. Which one of the following is the cumulative frequency of the entire data set
A)125 B) 24 C)40 D)118

100)Which one of the following is the mode value for the given data set
A)483 B) 125 C)400.5 D) 441

14
PYQ 2021 BSSEI (M-8777279548) Coaching for NET Geography

Nta NET Geography December 2021 (Shift-1)


1. Using the locational triangle', the concept of the least transport cost location was introduced by
(1) Walter Isard (2) M.K. Bandman
(3)D.M. Smith (4)Alfred Weber

2. Advocates of Humanistic geography among the following are :


(A)Harris (B) Blache (C )Guelke Ratzel
(D) Ratzel (E)Ley and Samuels
Choose the correct answer from the options given below:
(1) (B), (C) and (D) only
(2) (A), (B) and (C) only
(3) (A), (C) and (E) only
(4) (C) and (D) only

3.Given below are two statements : One is labelled as Assertion A and the other is labelled as Reason R.
Assertion (A): At greater ocean depths; water moves extremely slow and maintains a uniform low temperature.
Reasons (R): The oceanic surface returns heat and water to the lower atmosphere.
In the light of the above statements, choose the most appropriate answer from the options given below:
(1) Both (A) and (R) are correct and (R) is the correct explanation of (A)
(2)Both (A) and (R) are correct but (R) is NOT the correct explanation of (A)
(3) (A) is correct but (R) is not correct
(4)(A) is not correct but (R) is correct

4. Which of the following local winds are not related with drainage winds?
(1) Bora (2)Mistral (3)Santa Ana (4)Rossby

5.Who among the following geographers, focused on man centered Geography?


(A) Davis and Sample (B) Huntington and Blache
(C) Gefferron and Humboldt (D) Blache and Reclur
(E) Guelke and Jean Brunhes
Choose the correct answer from the options given below:
(1) (A) and (B) only (2)(B) and (D) only
(3) (D) and (E) only (4) (A) and (D) only

6.Match List I with List II :


List I Cultural Terms List II Definition
(A) Cultural Hearth I. Cultural parameter that specify learned behavior
(B) Cultural Landscape II. Areas of innovation from which key cultural elements diffused to
surrounding areas
(C)Cultural traits (III) Surfaces or areas modified by human action to produce tangible and
physical records over a period of time
(D) Cultural Realms (IV) Cultural regions showing related cultural complexes and landscapes
Choose the correct answer from the options given below:
1.(A)-(II). (B)-(IV), (C)-(I), (D)-(III) 2. (A)-(III). (B)-(II). (C)-(I), (D)-(IV)
3.(A)-(III). (B)-(IV). (C)-(1). (D)-(II) 4.(A)-(II), (B)-(III), (C)-(I), (D)-(IV)

7. Which one of the following is not a process of cultural change?


(1) Acculturation (2) Diffusion
(3) Innovation (4) Segregation

8. The most important factor responsible for localization of high-tech firms in silicon Valley of California
is……………….
(1) Cheap Labour (2) Access to the market
(3) Agglomeration Economy (4) High speed transportation

15
PYQ 2021 BSSEI (M-8777279548) Coaching for NET Geography
9.Arrange the given years in descending order on the basis of number of people affected by natural disasters
in India?
(A)1988 (B)1989 (C)1985 (D)1991 (E)1994
Choose the correct answer from the options given below:
(1) (A) ( C) (B) (D) (E) (2)(D) (A) (B) (C ) (E)
(3) (B) (A) (E ) (D) (4)(E ) (D) (B) (C ) (A)

10.Given below are two statements :


Statement I:Photographs are originally produced in analogue form.
Statement II: Image is produced in digital form.
In the light of the above statements, choose the most appropriate answer from the options given below:
(1)Both Statement I and Statement II are correct
(2)Both Statement I and Statement II are incorrect
(3)Statement I is correct but Statement II is incorrect
(4)Statement I is incorrect but Statement II is correct

11. Which one of the following group of states/U.T. in India recorded low sex ratio as per census 2011 data?
(1)Tamil Nadu, Uttarakhand, Himachal Pradesh, Haryana
(2) Jharkhand, Rajasthan, Odisha, West Bengal
(3) Manipur, Haryana, Goa, Uttar Pradesh
(4)Sikkim, Haryana, Jammu and Kashmir, Gujarat

12.Which of the following statements are correct regarding 'Doabs' of Punjab?


(A) Jalandhar Doab between the Beas and the Sutlej
(B)Bari Doab between the Beas and the Ravi
(C)Sindh Doab between the Ravi and the Jhelum
(D)Chhaj Doab between the Beas and the Ravi
Choose the most appropriate answer from the options given below:
(1)(A) and (B) only (2) (A) and (C) only
(3) (C) and (D) only (4)(B) and (D) only

13.In which one of the following water bodies the salinity is lowest?
(1)Great salt lake (2) Dead sea
(3)Lake Van (4) Gulf of Karabugas

14.Rally for valley' program in India was organized to highlight the problem of
(1) Environmental degradation (2) Biodiversity
(3)Resettlement of displaced persons (4)Loss of agricultural land

15.What is the total duration of time required to complete the two cycles of high and low tides at any place?
(1) 24 hours (2) 12 hours
(3)Less than 12 hours (4) More than 24 hours

16.Which one of the following is the correct chronological sequence of the German School of Geographers?
(A)F.V. Richthofen - F. Ratzel - Alfred Hettnen - Oscar Paschel
(B)F. Ratzel-Oscar Paschel - F.V. Richthofen - Alfred Hettner
(C )Alfred Hettner-Oscar Paschel-F.V. Richthofen-F. Ratzel
(D)Oscar Paschel - F.V. Richthofen - F. Ratzel - Alfred Hettner
(E)F.V. Richthofen - Alfred Hettner-F. Ratzel - Oscar Paschel
Choose the correct answer from the options given below:
(1) (A) and (D) (2)(C) and (D) (3) (A) and (B) (4) (D) only
17.Which one of the following refers to the mature stage of a thunderstorm?
(1)Electric tension between oppositely charged areas (2)Sudden onset of rain
(3)Absolute instability (4)Slackness of energy

16
PYQ 2021 BSSEI (M-8777279548) Coaching for NET Geography
18.Who among the following scholars coined the term permafrost?
(1)Ritter (2) Humboldt
(3)Agaziz (4) Ramsay

19.Which of the following states recorded literacy rates in descending order as per census 2011 data?
(A)Kerala (B)Himachal Pradesh (C)Haryana (D)Odisha
Choose the correct answer from the options given below:
(1)(A), D), (C), (B) (2) (B). (A). (C). (D)
(3) (A). (B). (C), (D) (4)(A). (C), (B), (D)

20.Match List I and List II:


List I(Proponent) List II(Philosophy of Geography)
(A) Gibson (1) Idealism
(B) Guelke (II) Realism
(C) Peet (III) Behaviourism
(D) Wright and Kirk (IV) Radicalism
Choose the correct answer from the options given below:
(1) (A)-(III), (B)-(II), (C)-(1), (D)-(IV) (2)(A)-(IV), (B)-(III), (C)-(II), (D)-(I)
(3)(A)-(II), (B)-(I), (C)-(IV), (D)-(III) (4)(A)-(1). (B)-(III). (C)-(IV), (D)-(II)

21.The phenomena of wind gap is associated with:


(1) Glacio-fluvial eroison Head ward erosion by river (2) Lateral erosion by river
(3) Head ward erosion river (4) Glacial planation

22. The per minute solar radiation received on one centimeter square at the outer surface of a plane is
(1) Convection Absorption (2) Solar constant
(3) Absorption (4) Reflection
23.Who among the following climatologists advocated Hekistothermal concept in his climatic classification?
(A)Miller (B)Koppen
(C )Candole (D)Trewartha
Choose the most appropriate answer from the options given below:
(1) (A) and (B) only (2)(B) and (C) only
(3)(C) and (D) only (4)(A) and (D) only

24.The San Andrean Fault is an example of


(1) Transform fault Spreading fault (2) Converging fault
(3) Spreading fault (4) Subduction fault

25.Consider the statements about currents in Indian Ocean


(A)North of equator, the formation of ocean currents is not influenced by the changing nature of the
monsoon wind system.
(B) Somali current is the warm current of Indian Ocean
(C )Somali current is the cold current of Indian Ocean
(D )Peru current is a cold current of Indian Ocean
(E)North-Equatorial current of Indian ocean disappears in the months
of August and September
Choose the correct answer from the options given below:
(1)(A) and (B) only (2)(C) and (E) only
(3)(B) and (©) only (4)(D) and (E) only

26.Lateral planation theory of pediment formation was propounded by


(1) W.M. Davis (2) Von Richthofen
(3)G.K. Gilbert(4) C. Darwin

17
PYQ 2021 BSSEI (M-8777279548) Coaching for NET Geography
27.Which one of the following are methods of load movement of sediment transportation in oceans?
(A)Traction (B)Saltation
(C)Turbulent flow (D)Flocculate
Choose the most appropriate answer from the options given below:
(1) (A) only (2)(A) and (B) only
(3)(A), (B) and (C) only (4)(B), (C) and (D) only

28. Which of the following statements are correct related to population studies?
(A) The Neo-Malthusian theories, contended that in human population fertility behavior is conditioned
by technological development
(B)Thomas Malthus was an English Economist and demographer
(C) Population cohort refers to, a group of population unified by a specific common characteristics
(D) Crude birth rate refers to birth rates minus death rates
(E) Population pyramids are constructed to identify natural growth rate of population
Choose the correct answer from the options given below:
(1) (A). (B), (D) only (2)(A). (C), (E) only
(3) (A), B). (C) only (4)(B), (D), (E) only

29.Which one of the following rivers is not a tributory of Yamuna river?


(1) Giri river (2) Tons river
(3)Bata river (4)Tangri river

30.Which of the following statements are incorrect about the origin and growth of international tourism
industry?
(A)Thomas Cook arranged the first package tours in 1841, taking travellers from Leicester to Lough
borough in U.K.
(B) The growing appreciation for variety in the cultural landscape in large European cities such as
Paris, Rome and London among tourist and other citizen played a role in expansion of tourism
(C ) The end of world war II provided the greatest impetus to tourism
(D ) Jet plane failed to herald the new era in International tourism
(E ) Boeing 747 in 1970 did not attract much international tourism
Choose the correct answer from the options given below:
(1) (A). (C) only (2) (C) (D) only
(3) (D). (E) only (4)(B), (D) only

31.Which one of the following statements was not considered by Malthus while stating Malthusian theory of
population growth?
(1)Food supply will increase in arithmetic proportion while population will increase in geometric
proportion
(2)With increasing population food scarcity will increase, resulting in high death rate of population
(3) Development of technology leading to increasing food production
(4)People will suffer from malnutrition resulting in high death rate

32.Read the statements carefully and find out the incorrect one, associated with GIS
(A)Data from several sources is integrated in consistent form
(B)'What if scenarios can be visualised
(C )In vector - data base, map data is stored using a matrix of grid cells
(D)Attribute data are descriptive data of features
Choose the most appropriate answer from the options given below:
(1)(A) only (2)(B) only
(3)(C) only (4)(D) only

18
PYQ 2021 BSSEI (M-8777279548) Coaching for NET Geography
33.Match List I with List II
ListI(Transport List II (Defination)
Network Measure)
A. Network Density i. Avertage Lenghth per Link
B. Inbox ii. Transport Network in terms of km of links (L) per square kilometer of
surface(S)
C. Eta Index iii. The relationship between total length of the graph (G)
D. Theta Index iv. Average amount of traffic per intersection
Choose The correct answer from the options given be;low
(1) A-I, B-II,C-III,D-IV (2)A-II, B-I,C-IV,D-III
(3)A-III,B-I,C-IV,D-II (4)A-II,B-III,C-I,D-IV

34. The theory of plate tectonics does not help to explain the origin and location of which one of the following?
(A)Mountain systems, major sea floors, earth quakes
(B) Earthquakes and major sea floors
(C) Ocean currents, climatic zones
(D)Earthquakes and mountain system Earthquakes
(E) Earthquakes
Choose the correct answer from the options given below:
(1)(A) and (B) only (2)(C) and (D) only
(3)(C) only (4)(A) and (E) only

35.Which of the following is the correct sequence of cosmic currents of Northern Pacific ocean
(A) West weind drift (B) NortH Equatorial current
(c) Kuroshio current (D) California current
Choose the correct answer from the options given below
(1) A B C D (2) C D B A
(3) B C A D (4) D A B C

36.Who among the following developed the concept of "Mental Map"?


(1) P.Gould, R. White and Lynch (2) Humboldt, Ritter and Paschel
(3) Semple and Huntington (4) Boulding and Hager

37.Match List I with List II:


List I(Theories of origin of tides) List II (Propounded by)
(A) Equilibrium Theory (1) Newton
(B) Dynamical Theory (II) Laplace
(C) Progressive wave Theory (III) Whewell
(D) Stationary wave Theory (IV) Harris
Choose the correct answer from the options given below:
(1)(A)-(IV), (B)-(III), (C)-(II), (D)-(I)
(B)(A)-(III), (B)-(II), (C)-(IV) (D)-(I)
(C) (A)-(II), (B)-(IV), (C)-(I), (D)-(III)
(D) (A)-(I), (B)-(II), (C)-(III), (D)-(IV)

38. The present day spatial distribution of Buddhism is highly concentrated among which of the following
group of regions?
(1) China, Tibet, Korea, Siberia (2) South East Asia, Tibet, Mongolia, Japan
(3) Northern India, China, South East Asia (4) Tibet, India, Middle East, Japan

39.In any thematic map, which of the following methods are used to record presence of phenomenon; its
spatial pattern, distribution and dispersion
(1)Isopleth Mapping (2) Isoline Mapping
(3)Choropleth Mapping (4) Dot method mapping

19
PYQ 2021 BSSEI (M-8777279548) Coaching for NET Geography
40.Which of the following pairs is not correctly matched?
(A)Kosi - Consequent river (B)Narmada - Rift Valley river
(C) Sutlej - Antecedent river (D)Ghaggar - Endoreic river
Choose the most appropriate answer from the options given below:
(1) (A) only (2) (B) only
(3)(C) only (4) (D) only

41. Who among the following conceived the idea of shipping canal between India and Srilanka?
(1) F. Luna LeopolD (2) L. Farrell
(3) A. D. Taylor (4) C. J. Barrow

42.Which one of the following group of State/UT recorded very high population growth rate during 2001-
2011 as per data given by census of India?
(1) Andhra Pradesh, Meghalaya, Himachal Pradesh Tripura
(2)Mizoram, Sikkim, Goa, Bihar
(3)Bihar, Jammu and Kashmir, Goa, Delhi
(4) Bihar, Jammu and Kashmir. Meghalaya, Mizoram

43. Match List I with List II


List I Whitlesey's Agricultural region List II Name of country
(A) Commercial dairy farming (1) Mangolia
(B) Commercial grain farming (II) Sri lanka
(C) Nomadic herding (III) New Zealand
(D) Commercial plantation (IV) Argentina
Choose the correct answer from the options given below:
(1) (A)-(1), (B)-(II), (C)-(III), (D)-(IV)
(2)(A)-(II). (B)-(III). (C)-(IV), (D)-(I)
(3)(A)-(III). (B)-(IV), (C)-(I), (D)-(II)
(4)(A)-(IV). (B)-(III), (C)-(II), (D)-(1)

44.Indian States are arranged in ascending order on the basis of road length (2017)
(A)Tamil Nadu (B)Karnataka
(c) Maharasthra (D)Gujarat
Choose the correct answer from the options given below:
(1) (C), (B), (A), (D) (2)(A), (B). (C), (D)
(3)(B). (C), (D). (A) (4) (D). (B), (A),(C)

45.Identify the processes of vorticity :


(A)Acceleration (B)Frictional influence
(c) Roughness (D)Rotation (E)Angular velocity
Choose the correct answer from the options given below:
(1 ) A B only (2) C D ONLY
(3) D C only (4) A E only

46. Geography in 17 Volume was authorized by


(1) Ptolemy (2) Ritter (3) Strabo (4) Aristotle

47.Which of the following statements are correct about the benefits of international trade?
(A) It has allowed an enormous variety of resources to enter into trade basket
(B)It facilitates the distribution of a wide range of manufactured goods, produced in different parts of the
world to global market
(C) International trade decreases the overall costs of production
(D)It has helped the developing countries more than developed countries
(E) It has helped the consumers to buy more goods from whatever wages they earn and their living standards
improved

20
PYQ 2021 BSSEI (M-8777279548) Coaching for NET Geography
Choose the correct answer from the options given below:
(1) (A). (B), (E) only (2) (C), (D), (E) only
(3) (A), (B), (C) only (4) (A), (C). E) only

48.Match List I with List II :


List I Land forms List II Agents of gradation
A)Monadnocks Wind
(B) Blind valley (II) Glaciea
(C) Drumlins (III) River
(D) Zeugen (IV) Ground water
Choose the correct answer from the options given below:
(1)(A)-(II), (B)-(III). (C)-(I), (D)-(IV)
(2)(A)-(IV). (B)-(II), (C)-(III), (D)-(1)
(3)(A)-(I). (B)-(II), (C)-IV), (D)-(III)
(4) (A)-(III), (B)-(IV). (C)-(II), (D)-(I)

49. Which one of the following indicates a population pyramid with wide base narrowing as the age Cohorts
progress?
(1)High fertility rates and low mortality rates
(2)Low fertility and low mortality rates
(3) High fertility and high mortality rates
(4) High fertility and high age specific fertility rates

50.Which one of the following term is used for the distance between trough to crest of a wave?
(1) Wave trough (2) Wave velocity
(3)Wave height (4) Wave period

51.Match List I with List II :


List ITropical Disturbances List II Wind speed in circulation (in kmph
(A) Low pressure area (I) 89- 118
(B) Cyclonic storm (II) 31 - 49
(C) Depression (III) <31
(D) Severe cyclonic storm (IV) 62-88
Choose the correct answer from the options given below:
(1)(A)-(IV), (B)-(II), (C)-(III), (D)-(1)
(B ) (A)-(III). (B)-(II). (C)-(I). (D)-(IV)
(C )(A)-(III), (B)-(IV), (C)-(II), (D)-(I)
(D )(A)-(I). (B)-(IV), (C)-(III), (D)-(II)

52.Which of the following terms are not associated with sling psychrometer?
(A)Thermometer (B)Swivel joint (C)Screen (D)Sieve
Choose the most appropriate answer from the options given below:
(1) (A) and (B) only (2)(B) and (C) only
(3) (C) and (D) only (4) (B) and (D) only

53.Match List I and List II :


List I Urban terms List II Definitions
(A) Urbanized Area (1) Continuously built up landscapes with no political boundaries
(B) Conurbation (II) Continuous urban built-up area with multiple urban centres
(C) Urban Influence Zone (III) A city with much higher population(more than twice) than the
proportion of second largest city
(d) Primate city (IV) Areas outside cities having significant city influence
Choose the correct answer from the options given below:
1. A-I, B-II, C-IV, D-III 2. A-III,B-II,C-I,D-IV
3.A-II,B-III,C-I,D-IV 4.A-IV,B-II,C-I,D-III

21
PYQ 2021 BSSEI (M-8777279548) Coaching for NET Geography
54.Arrange the following in ASENDING ORDER ON THE BASIS OF PERCENTADE OFR TRIBAL
POPULATION(2011)
(A)Madhya Pradesh (B) Maharasthra
(C) Rajasthan (D) Meghalaya
Choose the correct answer from the options given below:
1. A B C D 2. C D B A
3.B C A D 4. D A B C

55. Arrange the following in a spatial order from equator


(A)Northeast trades (B)Horse latitudes
C)Doldrum (D)Westerlies
Choose the correct answer from the options given below:
(1)(C), (A), (B), (D) (2)(B). (C), (D). (A)
(3)(A), (B) (C), (D) (4 )(D). (A), (C), (B)

56.Which one of the following groups of land forms is produced by erosion?


(A)Fiords, Drumlins, Natural levees
(B)Playa, Pinacle, Swallow holes
(c ) Esters, Outwash plain. Till
(D) Cirques, Stalagmites, Ventifacts
Choose the most appropriate answer from the options given below:
(1) (A) and (D) only (2) (A) only
(3) (B) and (D) only (4) (B) only

57.Identify the cryogenic indicators of pale climates :


(A) Evaporite deposites (B) Ice cores (C) Duricrusts
(D) Varves (E) Ice sheets
Choose the correct answer from the options given below:
(1)(A) and (B) only (2) (B) and (E) only
(3)(A) and (C) only (4)(C) and (D) only

58.Which one of the following statements is/are incorrect regarding waves?


(A)wave height is not indicator of wave energy
(B)wave height is controlled by wind strength
(c)the distance of open water a wave travels over affects the wave height
(D)the depth of the sea is important factors affecting wave height
Choose the most appropriate answer from the options given below:
(1) (A) only (2)(A) and (B) only
(3) (C) only (4) (C) and (D) only

59.Which of the following natural characteristics are associated with dry monsoon forests of India?
(A)Annual rainfall is below 50 cm
(B)The trees have short roots
(C)Conditions remain favourable for plant growth throughout the year
(D)Thorny shrubs and grasslands are found between the trees
(E) Mango, Mahua, Sisam. Keekar etc, are the prominent trees
Choose the correct answer from the options given below:
(1) (A) and (B) only (2)(C) and (D) only
(3) (B) and (C) only (4) (D) and (E) only

60.Arrange the following with respect to the interior of a thunderstorm cell


(A)Downdraft (B) Rain (C)Storm travel (D ) Anvil top
Choose the correct answer from the options given below:
(1) (C), (D), (A), (B) (2)(B), (C), (D). (A)
(3)(A). (B), (C). (D) (4)(B). (C), (A) (D)

22
PYQ 2021 BSSEI (M-8777279548) Coaching for NET Geography
61. Which of the following is the correct sequence in the development of Geographic Thought?
(A) Greeks, Roman, Arabs, Dark Ages
(B) Greeks, Roman, Dark Ages, Arabs
C) Roman, Greeks, Arabs, Dark Ages
(D) Arabs, Greeks, Roman, Dark Ages
(E) Greeks, Dark ages, Roman, Arabs
Choose the correct answer from the options given below:
1. B Only 2. A only 3. C ONLY 4. E Only

62.Who among the following scholars examined the regional and geographical implications of growth pole
theory earlier propounded by perroux?
(1) Gunnar Myrdal (3)AO. Hirschman
(2) John Friedmann (4)J.Boudellie

63.In four stages of demographic transition theory, the stage in which birth rate continues to be high but death
rate declines rapidly, is :
(1) Stage 4 (2) Stage 1
(3) Stage 2 (4) Stage 3

64.Which one of the following industries generates highest amount of waste water in the Ganga in U.P.?
(1)Chemical (2)Pulp and Paper
(3)Sugar (4)Distillery

65.The first meteorological satellite was


(1) TRMM (2) TIROS-I
(3) MODIS (4) NOAA

66Arrange the following layers of vegetation on the basis of their height in ascending order
(A) Dominant layer (B) Herbaceous layer
(C) Codominant layer (D) Ground layer
Choose the correct answer from the options given below:
(1) (B). (C). (A), (D) (2) (D). (C). (B). (A)
(3)(A). (B). (C). (D) (4)(D). (B ) (C ) (A)

67.Population migration
(A) pull factor in population migration refers to availability of livelihood opportunities at destination
places
(B) step migration refers to stepwise availability of livelihood opportunities at destination places
(C) push factor in population migration refers to less limited availability of livelihood opportunities at
originating place
D) migration field refers to return migration to originating place
Choose the most appropriate answer from the options given below:
(1) (B) and (D) only (2) (A) and (B) only
(3)(B) and (C) only (4) (A) and (C) only

68.Match List I and List II :


List I List II
(Thinker/Scholar) (Theory/Model/Strategy)
(A) Gunnar Myrdal (1) Strategy of unbalanced growth
(B) A.O. Hirschman (II) Circular and cumulative causation
(C) John Friedmann (III) Growth centre and growth axis
(D) J. Boudeville (IV) Core and periphery
Choose the correct answer from the options given below:
(1)(A)-(I). (B)-(II). (C)-(III), (D)-(IV) (2)(A)-(II). (B)-(1). (C)-(IV), (D)-(III)
(3)(A)-(III). (B)-(IV). (C)-(I), (D)-(II) (4)(A)-(IV), (B)-(III), (C)-(II), (D)-(I)

23
PYQ 2021 BSSEI (M-8777279548) Coaching for NET Geography
69.Given below are two statements:
Statement I:Highest source of marine pollution is shipping.
Statement II:The South-East Asia has highest percentage of endangered coral reefs by marine pollution.
In the light of the above statements, choose the correct answer from the options given below:
(1)Both Statement I and Statement II are true
(2)Both Statement I and Statement II are false
(3)Statement I is true but Statement II is false
(4)Statement I is false but Statement II is true

70.Which one of the following is incorrect about agro-industrialization –


(1)Needs large agricultural farms
(2)Low labour input per unit of production
(3)Highly qualified managers
(4)Less use of chemical fertilizers and pesticides

71.Which one of the following methods is used by researchers for reducing 'biasness of scale while
compositing several indicators?
(1)working out percentages and adding them together
(2)adding actual values of all indicators
(3) segregating positive and negative indicators
(4) dividing the values of indicators with their mean values

72. A researcher wants to study the relationship of family size to income. He/She classifies population into
different income slabs and then takes a random sample from selected slabs. Which one of the following
techniques is he/she working with?
(1) Cluster sampling (2) Random sampling
(3)Stratified random sampling (4) Systematic sampling

73.Identify the terrestrial disasters


(A)Cold wave (B)Seismic
(C)Collision of meteors with the earth (D)Tsunami
Choose the correct answer from the options given below:
1. A & B 2. B C 3. C D 4. B D

74. Which of the following statements about international trade in post-globalization phase are correct?
(A)The share of international trade has doubled now in comparison to that of 1950
(B)Protectionist tendency in trade stands highly discouraged now
(C) The share of developing countries has increased substantially
(D)The share of manufactured goods has grown sharply
Choose the most appropriate answer from the options given below:
(1) (A), (B), (D) only (2)B). (C). (D) only
(3) (A), (C). (D) only (4)(A), (B), (C) only

75.Match List I with List II:


List I(Soil Type) List II(Places of occurrence)
A) Laterite (1) Rahr plain
(B) Saline (II) Maharasthras plain
(C) Black (III) Rajmahal Hills
(D) Red loam (IV) Kutch region
Choose the correct answer from the options given below:
(1)(A)-(ID). (B)-(IV). (C)-(III). (D)-(1)
(2)(A)-IV). (B)-(II). (C)-(III), (D)-(1)
(C )(A)-(III). (B)-(IV). (C)-(II). (D)-(I)
(D )(A)-(IV). (B)-(II), (C)-(I), (D)-III

24
PYQ 2021 BSSEI (M-8777279548) Coaching for NET Geography
76. Given below are two statements : One is labelled as Ascertain A and the other is labelled as Reason R.
Assertion (A): Value system is a commonly held set of beliefs, understandings and controls, that unite
members of a culture group.
Reasons (R): Polytheism adheres to belief in many religious beliefs.
In the light of the above statements, choose the correct answer from the options given below:
(1)Both (A) and (R) are true and (R) is the correct explanation of (A)
(2)Both (A) and (R) are true but (R) is NOT the correct explanation of (A)
(3)(A) is true but (R) is not false
(4)(A) is false but (R) is true

77.Stop and Go determinism was propounded by


(1) F. Ratzel (2) G. Taylor
(3) A.V. Humboldt (4)J. Brunhes

78.The volume, nature and direction of global trade is influenced by


(A) Economic advancement
(B)Comparative and locational advantage
(C) Historical factors and investment
(D) Changes in global market
(E) Geographical distance
Choose the correct answer from the options given below:
(1) (A) and (B) only (2) (B). (C) and (E) only
(3)(B), (C) and (D) only (4) (A), (B) and (C) only

79.Belize barrier reef is located between:


(1) Mexico and Cuba (2) Jamaica and Honduras
(3) Mexico and Guatemala (4) Mexico and Bahamas

80.Which of the following is a renewable resource?


(1) Metal ores (2) Natural gas (3) Fuel Cells (4)Fishes

81.Geo-Strategic model in political geography was propounded by:


(1)Spytman (2)Saul Cohen
(3)John Short (4)John Agnew

82.Given below are two statements : One is labelled as Ascertain A and the other is labelled as Reason R.
Assertion (A):Prices and demand of real estate increases as distance towards CBD reduces
Reasons (R):Central Business District (CBD) of a city has high concentration of retail store offices and cultural
and recreation activities.
In the light of the above statements, choose the most appropriate answer from the options given below:
(1)Both (A) and (R) are correct and (R) is the correct explanation of (A)
(2) Both (A) and (R) are correct but (R) is NOT the correct explanation of (A)
(3) (A) is correct but (R) is not correct
(4)(A) is not correct but (R) is correct

83.Singularities pertaining to recurrence of anticyclonic weather are associated with which area of the globe?
(1) South Asia (2) North America
(3) Britain (4) Australia

84.Which of the following waterfalls pertain to Narmada River?


(A) Mandhar (B)Hiran (c )Tawa (D) Dardi (E)Sahasradhara
Choose the correct answer from the options given below:
(1) (A), (B) and (C) only 2.(A), (D) and (E) only
3.(B), (C) and (D) only 4. (C). (D) and (E) only

25
PYQ 2021 BSSEI (M-8777279548) Coaching for NET Geography
85.Which one of the following frequency distribution is negatively skewed?

(1)A (2)B 3. C (4)D

Mean

86. Which one of the following depicts correct demographic equation of population growth?
(1)Natural increase and dependency ratios
(2) Natural increase and net migration
(3) In-migration and birth rate
(4) Out-migration and death rate

87.Who among the following formalized theconcept that there exists relationship between the spatial
organization and National development?
(1)Friedmann (2) Dickens Kuklinski
3. Kuklinski (4)Luten

88. Consider the following and choose the correct sequence of maps from larger scale to smaller scale
(A)Cadastral (B) Topographical
(C)Wall (D) Globe
Choose the correct answer from the option given below:
(1) (A), (C), (D), (B) only (2)(B),(A), (C), (D) only
(3) (A), (B), (C), (D) only (4)(C), (A). (B), (D) only

89. Which of the following are very high risk areas of earthquake?
(A)Tripura (B ) North Bihar
(C )Eastern Gujarat (D)Haryana
Choose the most appropriate answer from the options given below:
(1) (A) and (B) only (2) (B) and (C) only
(3)(C) and (D) only (4)(A) and (D) only

90.
SL NO. TEMPERATURE NEMBEDR OF STATION
1 0-10 1
2 10-20 2
3 20-30 3
4 30-40 4
5 40-50 5
6 50-60 6
7 60-70 7
8. 70-8- 8.
9. 80-90 9
Which one of the following is the median of the above data?
(1) 45°C (2) 35°C (3) 55°C (4) 400 C

91.The number of stations that recorded temperature less than 40°C:


(1) 3 (2) 9 (3) 10 (4) 5

26
PYQ 2021 BSSEI (M-8777279548) Coaching for NET Geography
92. The Third Quartile (Qa) of the distribution is:
(1) 45°C (2) 49.375°C (3) 45.5°C (4) 59.375°C

93.The average temperature in the region is:


(1) 40°C (2) 45°C (3) 50°C (4)55°C

94What percent of stations recorded temperature between 30°C to 70°C?


(1) 40 (2) 54 (3) 62 (4) 64

95.Geography as a discipline is focused on understanding the relationship between Man and Nature. It is built
on experiences of successive generations. The history of geography is basically concerned with the
development of human consciousness about the possibilities and limitations that nature offers for human
growth. The word geography was first used by the Greek scholar Eratosthenes in the third century. Hartshorne
said that Geography provides rational description of the earths surface above and below depending on the
progress in technology. The study of man is the realm of humanities, where as science focused on the study
of nature. Geography studies them simultaneously.
However geography as a university level discipline started only in the ninetenth century. Humboldt and Ritter
demonstrated the usefulness of Geography as an universal science. After this period many scholars like Ratzel
and Richtofen in Germany. Blache and Reclus in France projected it as an integrated study. Man, Nature
relationship, hormony with nature led to concepts of Determination and possibilism, however kant tried to
address this confusion to a certain extent. With the emergence of different schools of thought Dualism and
Dichotomies in Geography like general versus regional - physical versus human - Historical versus
contemporary - Nodal versus regional concepts developed; and prevailed. The advent of quantitative
revolution had further diversified the discipline. Paradigm, systems approach and models in geography
became significant aspects of modern geographic thought. Paradigm proposed by Kuhn had references on
areal differentiation, exceptionalism, spatial analysis. Models in Geography made significant progress.
Modern Themes in Geographic thought like positivism, pragmatism, functionalism etc work introduced.
The concepts of Determinism and positivism emerged as a result of integrated Hinduism by
(1) Greek and Roman (2) Greek and Arabs
(3)Roman and Arab (4) German and French

96. Which of the following is the recent development in Geographic studies?


(1)Regional geography (2) Astronomy
(3) Determination (4) Positivism

97.Who were the first to use the word Geography?


(1)Arabs (2)Roman (3)Greeks (4)Chinese

98. Which major development in Geographic studies during the recent period changed the direction in its
growth?
(1) Paradigm (2) System Approach
(3) Quantitative Revolution (4) Models

99. The usefulness of Geography as an Universal Science was demonstrated by


(1) Eratosthenes and Strabo (2)Hecataeus and Hipparchus
(3)Humboldt and RitteR (4)Aristotle and Eratosthenes

27
PYQ 2021 BSSEI (M-8777279548) Coaching for NET Geography

Nta NET Geography December 2021 (Shift-2)


1. As the wave approach coast, the wave form changes radically. Which one of the following is correct
sequence of changes?
(A)Closely spaced crests
(B)Wave breaker Swash Back wash
(C)Swash
(D) Back wash
Choose the correct answer from the options given below:
(1) (B). (C). (A), (D) (2) (A), (D), (C), (B)
(3)(C). (B). (D). (A) (4)(A). (B). (C). (D)

2.Who is usually given the credit for establishing the validity of the idea that there was an Ice Age during
which ice sheets covered much of the Northern Europe?
(1) Jukes (2) Agassiz
(3) Ramsay (4)W. Penck

3.Which one of the following is closely associated with westerlies?


(A)Furious fifties (B)Boyle's gas law
(C)Walker circulation (D)Rossby waves
Choose the most appropriate answer from the options given below:
(1)(A) and (B) only (2) (B) and (C) only
(3)(C) and (D) only (4)(A) and (D) only

4.Cirques are landform of


(1) First order (2) Second order
(3)Third order (4)Fourth order

5.The agricultural activities from the city centre to the periphery propounded by Von Thunen: Arrange it
(A)Grain farming (B)Market Gardening
(C)Three fold system (D)Firewood and lumbering product
Choose the correct answer from the options given below:
(1) (D). (B), (A), (C) (2)(A). (B), (C ). (D)
(3)(C), (D). (A). (B) (4) (B). D). (A). (C )

6.Match List I with List II :


List I List II
(A) Culture Culture traits that are functionally
interrelated
(B)Cultural Complex (II) Area that is distinct from surrounding areas in specific
characteristic
Cultural Region Specialized behavioual pattern under
standings and adaptation of a group
(D) Cultural Integration (IV) Interlocking nature of the sociological technological and
ideological subsystem
Choose the correct answer from the options given below:
(1) (A) - (III). (B) - (IV), (C) - (II). (D) - (1)
(2) (A) - (III). (B) - (I). (C) - (II). (D) - (IV)
(3) (A) - (1). B) - (II). (C) - (III). (D) - (IV)
(4)(A) - (TV). (B) - (III), (C) - (II). (D) - (1)

7.The economic zone' of India from the coastline extends upto a distance of
(1)112 nautical miles (2)120 nautical miles
(3)200 nautical miles (4)320 nautical miles

28
PYQ 2021 BSSEI (M-8777279548) Coaching for NET Geography
8.Which of the following map scales would be most useful for study of the details of geographic features in
the landscape of farming community
(1) 1: 3.000.000 (2) 1:1.000.000
(3) 1: 10.000 (4) 1: 125.000

9.In concentric zone model, the correct order of zones from city core to periphery is
(A)Low-Class residential (B)Whole sale light manufacturing
(C) CBD (D) Medium-class residential
Choose the correct answer from the options given below:
(1) (A). (B), (C), (D) (2)(B). (C), D), (A)
(3) (C). (B). (A) (D)(4) (D). (B). (C). (A)

10.Which one of the following in the correct sequence in the development of German school of thought?
(A)Ratzel-Paschal-Hettner- Troll (B)Paschal-Ratzel-Hettner-Troll
(C)Ratzel-Troll-Paschal-Hettner (D)Troll-Paschal-Hettner-Troll
(E)Hettner-Paschal-Troll-Ratzel
Choose the correct answer from the options given below:
(1) (A) (2) (B) (3) (C ) (4)(D)

11.What will happen if the earth absorbs all the incident energy?
(1)Constant heating up (2) Constant cooling up
(3) Constant transmission (4) Constant reflection

12.According to Koppen's climatic classification 'Amw' climate is found in


(1) Assam (2) Goa
(3) Punjab (4) Arunachal Pradesh

13.Northward slopes remain forested in mountainous areas due to


(1) Radiation intensity (2) Sensible heat
(3) Diffusion of heat (4) Rayleigh scattering

14. Which one of the following exert maximum tide producing force on the ocean water?
(1) Sun (2) Moon (3) Venus (4) Mars

15.Given below are two statements : One is labelled as Assertion (A) and the other is labelled as Reason (R)
Assertion (A):Among the states of India Madhya Pradesh recorded highest percentage of Scheduled Tribe
population in census 2011.
Reason (R): Madhya Pradesh is located in central part of India dominated by large area under forests with
least road connectivity and accessibility.
In the light of the above statements, choose the most appropriate answer from the options given below:
(1)Both (A) and (R) are correct and (R) is the correct explanation of (A)
(2)Both (A) and (R) are correct but (R) is NOT the correct explanation of (A)
(3)(A) is correct but (R) is not correct
(4)(A) is not correct but (R) is correct

16.Arrange the given regions/states in ascending order of wind and cyclone damage risk zone
(A)Eastern Coastal plain (B)Rajasthan
©Most part of Maharashtra (D)Karnataka (except coastal area)
Choose the correct answer from the options given below
(1) (A), (C). (B), (D) (2)(C). (B). (A). (D)
(3) (D), (C), (B), (A) (4)(B). (C), D). (A)

17.Which of the following vegetation species do not pertain to Tropical Thorn Forests?
(A) Nirmali (B) Khejra (C)Tendu (D) Haldu (E) Babul

29
PYQ 2021 BSSEI (M-8777279548) Coaching for NET Geography
Choose the correct answer from the options given below:
(1) (A) and (B) only (2) (C) and (D) only
(3) (A) and (E) only (4)(B) and (C) only

18.Environmental determinism was criticised and opposed by


(A) Huntington on and Semple (B)Vidal de la Blache and L Febvre
(C)Humboldt and Ratzel (D) Ritter and L Febvre
(E) Blache and Reclus
Choose the correct answer from the options given below:
(1) (A) and (B) only (2) (B) and (C) only
(3) (D) and (E) only (4) (B) and (E) only

19.The sea plains are almost absent between:


(1) 20°S-30°S Latitudes (2) 60°N - 70°N Latitudes
(3)40°S - 50°S Latitudes (4)30°S-40°S Latitudes

20.Which of the following statements are incorrect about the growth of tourism industry in the world?
(A) Steady growth in real incomes of the people in the world has contributed to growth ofTourism
(B) Increased desire of the people to experience different cultures and landscapes hascontributed to
tourism growth over time
(C) Many countries have heavily invested in development of tourism infrastructure toencourage
tourism
(D) There are little range of destinations open for the middle-income group travelers
(E) People from developing countries now dominate the global tourism
Choose the correct answer from the options given below:
(1) (A), (B) only (2)(B). (C) only
(3)(D), (E) only (4)(C). (D), (E) only

21.Who among the following were the believers of Environmental Determinism?


(A) C. Darwin (B) Humboldt (C) Ratzel
(D) Semple (E) Blache
Choose the correct answer from the options given below:
(1) (A), (C) and (D) only (2)(A). (B) and (E) only
(3) (A) and (B) only (4) (A) and (E) only

22.State/UTwise location of Biosphere Reserves in India.


(A) Nokrek in Meghalaya
(B) Cold Desert in Jammu and Kashmir
(C) Simlipal in Odisha
(D) Seshachalam in Tamil Nadu
Choose the correct answer from the options given below:
(1)(A) and (B) only (2)(B) and (C) only
(3) (A) and (C) only (4) (B) and (D) only

23. Match List I with List II :


List IArab Geographer List II Books
(A) Al-Masudi (1) Kitab al Hind
(B) Al-Idrisi (II) Muqaddimah
(c ) Al-Biruni (III) Kitab Muruj-al-Dhahab
(D) Ibn-Khaldun (IV) Tabula Rogeriana
Choose the correct answer from the options given below:
(1) (A) - (III). (B) - (IV). (C) - 1). D) - (II)
(2) (A) - (1). (B) - (IV), (C) - (III), D) - (II)
(3) (A) – (II). (B) - (III). (C) - (IV). D) - D
(4) (A) – (IV), (B) - (1), (C) - (II), (D) – (III)

30
PYQ 2021 BSSEI (M-8777279548) Coaching for NET Geography
24.The term "Anthropogeographic" was coined by Ratzel in his work
(1) Study of harmonious reciprocal relation of man and nature
(2) Geography of man in terms of individuals and races
(3) Geography of man and his work in relation to carth surface
(4) Organic theory of society and state

25.Match List I with List II:


List I Relief Features List II Name of Oceans
(A) Puerto Rico deep (1) Pacific Ocean
(B) Sunda Trench (II) Atlantic Ocean
(C) Philippine trench (III) Indian Ocean
(D) Point Barrow (IV) Arctic Ocean
Choose the correct answer from the options given below:
(1) (A) - (1), (B) - (II), (©) - (III), (D) - (IV)
(2)(A) - (TV). (B) - (III). (C) - (II), (D) - (1)
(3)(A) - (II). (B) - (III). (C) - (I). (D) - (IV)
(4)(A) - (III). (B) - (IV). (C) - 1). (D) - (II)

26.The line of equal distribution at 45° used for measuring inequalities is called
(1) Scatter diagram (2) Histogram
(3) Lorenz curve (4) Ogive curve

27.Which one of the following are considered while preparing future population projections?
(1)Crude death rates and their temporal changes
(2) Rate of population increase and future assumptions of death and birth rates based on past
data
(3) General fertility rates and their temporal changes
(4)Age and sex composition rates and their temporal changes

28.Match List I with List II :


List I Spectral Band List II Principal Applications
(A) Blue (1) Vegetation Stress Analysis
(B) Green (II) Differentiating soil from vegetation
(0) Red (III) Coastal Mapping
(D) Thermal (IV) Vegetation discrimination
Choose the correct answer from the options given below:
(1) (A) - (1). (B) - (II) (0) - (III). (D) - (IV) (2) (A) – (III). (B) - (IV). (C) - (II), (D) - (1)
(3) (A) - (II). (B) - (IV). (C) - (I). (D) - (III) (4) (A) - (IV). (B) - (III), (C) - (II), (D) - (1)
𝑌 𝑇
29. Who among the following scholars applied the formula PI= 𝑌𝑛 / where PI=Agriculturalproductivity
𝑇𝑛
index: Y = Total production of selected crops in unit area; Y, = Total production of the same selected crops
in the country; T = Total cropped area of the unit;
T = Total cropped area in the country for measuring agricultural productivity level.
(1) Bhatia (2) Enyedi (3) Weaver (4) Shafi

30. Which is not true regarding mountain climate of India?


(A)Sharp contrasts in temperature between the sunny and shaded slopes
(B)Diurnal range of temperature is high
(c)Uncertain character of rainfall with drought proneness and crop failures
(D)High temperatures accompanied by high humidity
Choose the most appropriate answer from the options given below:
(1) (A) and (C) only (2) (B) and (C) only
(3) (A) and (B) only (4)©) and (D) only

31
PYQ 2021 BSSEI (M-8777279548) Coaching for NET Geography
31. Which one is the correct ascending order of satellite sensors in terms of higher spatial resolution?
(A)RBV of Landsat (B)TM of Landsat
(C)LISS II of IRS-1C (D)PAN of IKONOS-2
Choose the correct answer from the options given below:
(1) (A), (D), (C). (B) (2)(A). (C), (B), (D)
(3) (D). (A). (B). (C) (4) (B). (C), (A), (D)

32.Save the Silent Valley' movement, a social movement to save the tropical green forest from destruction for
a regional development project was territorially based in Indian State of –
(1) West Bengal (2) Bihar
(3) Kerala (4) Gujarat

33.Laminar flow of air refers to


(1)Movement of particles in all direction
(2) Movement of particles in one direction
(3) Deflection in the direction of surface winds
(4) Decrease of air pressure between two isobars

34.In which of his book in 1919, Mackinder rechristened his concept of 'Pivot Area' as 'Heart land.
(1) The Round world and the winning of peace
(2)Foreign Affairs
(3)World war and Geography
(4)The Democratic ideals and Reality

35. Arrange the four Indian oceanic relief features in order of their location from north to south direction
(A)Lacca dive-Chagos ridge (B)Chagos-Saint Paul ridge
(C)Amsterdam-St. Paul Plateau (D)Kergelen-Gaussberg ridge
Choose the correct answer from the options given below:
(1) (D). (C). (B). (A) (2)(C). (B), (A), (D)
(3) (A), (B). (C), (D) (4)(B). (C), (A), (D)

36.Which one of the following statements are correct?


(A)Ocean currents provide a high degree of regulation to the thermal environments of the earth's
surface
(B)On a local scale, warm water currents bring moderating influence to coasts in arctic latitudes
(C)Cold currents are not able to bring down the temperature of tropical deserts
(D)Density differences may also cause flow of ocean water
Choose the correct answer from the options given below:
(1) (A). (C) and (D) only (2)(A). (B) and (D) only
(3)B. (C) and (D) only (4)(A), (B) and (C) only

37.Which of the following mines have their association with gypeum?


(A)Nagaur (B)Lapsa Buru (C) Bhadwah (D)Ghagidith (E) Jasmar
Choose the correct answer from the options given below:
(1) (A), (B) and (C) only (2) (B), (C) and (D) only
(3)(C), (D) and (E) only (4)(A) (C) and (E) only

38.A Free Trade Zone, in international trade, happens to be there, where


(1)All the participating member countries impose a common tariff on imports from non-member
countries
(2) All the participating countries allow free movement of labour and capital
(3)Member countries abolish tariff and quotas on trade but maintain independence to impose
restrictions on imports from non-member countries
(4) Member countries adopt common economic policies on issues such as agriculture, transport,
industry and regional policy

32
PYQ 2021 BSSEI (M-8777279548) Coaching for NET Geography
39.Match List I with List II :
List I Hydro power units List II State
(A) Dehar I.Karnataka
(B) Srisailam II.Karnataka
(C) Kalindi (III) Andhra Pradesh
(D) Sharavathy (IV) Himachal Pradesh
Choose the correct answer from the options given below:
(1)(A) – (I), (B) - (IV), (C) - (III). (D) – (II)
(2)(A) - (TV), (B) - (II), (C) - (III). D) - (1)
(3)(A) - (IV), (B) - (III), (C) - (II). D) - (1)
(4)(A) - (III), (B) - (II). (C) - (IV), (D) - (1)

40.Which of the following is not associated with structure of a tropical cyclone?


(1) Eye Wall (2) Spiral Bands
(3) Annular Bands (4) Ekman Spirals

41.Given below are two statements : One is labelled as Assertion (A) and the other is labelled as Reason (R)
Assertion (A):According to the 2011 census, the scheduled tribes accounts for 8.6 percent of India's population
and are spread largely in forested and hilly regions.
Reason (R):The essential characteristics of scheduled tribe communities are primitive traite, geographical
isolation and economic backwardness.
In the light of the above statements, choose the correct answer from the options given below:
(1)Both (A) and (R) are true and (R) is the correct explanation of (A)
(2)Both (A) and (R) are true but (R) is NOT the correct explanation of (A)
(3)(A) is true but (R) is false
(4)(A) is false but (R) is true

42.Match List I with List II:


List I Goals of List II( Items )
National Water
Mission
(A) Goal - 1 1) Basin level integrated water resource management
(B) Goal - 2 II Attention to vulnerable areas
(0) Goal - 3 (III) Water conservation by promotion Of citizen and state actions
(D) Goal - 5 (IV) Comprehensive water data base
Choose the correct answer from the options given below:
(1) (A)-1). (B) - (II), (C) - (III). (D) - (IV)
(2)(A) - (II). (B) - (IV), (0) - (1). (D) - (III)
(3)(A) - (III). (B) - (II). () - (1). (D) - (IV)
(4)(A) - (IV). (B) - (III). (C) - (II) (D) - (1)

43. Given below are two statements:


Statement I: All human actions associated with production, marketing and consumptionof goods and
services are defined as economic activities.
Statement II: All human actions linked not only with production, marketing andconsumption of goods
and services but the time spent on leizure and pleasure activities are also defined as economic activities.
In the light of the above statements.
choose the most appropriate answer from the options given below:
(1)Both Statement I and Statement II are correct
(2) Both Statement I and Statement II are incorrect
(3) Statement I is correct but Statement II is incorrect
(4) Statement I is incorrect But Statement II is correct

33
PYQ 2021 BSSEI (M-8777279548) Coaching for NET Geography
44. Which of the following statements are correct about sea water.
(A)The density of sea water depends upon temperature, pressure and salinity
(B)Convergence of water has no effect on density
(c)The thermal expansion of water varies with the increase and decrease of temperature
(D)In the middle latitudes, due to an excess of precipitation the density is higher
Choose the correct answer from the options given below:
(1) (A) and (D) only (2) (A) and (C) only
(3) (B) and (C) only (4)C) and (D) on

45.Which of the following faunal groups has the highest percentage of endemic species?
(1) Amphibians (2) Birds
(3) Insects (4) Reptiles

46.The most accurate representation of culture units are given below. Arrange them from highest to lowest
(A)Complex (B)Realm
(C) Region (D )Trait
Choose the correct answer from the options given below:
(1) (B). (C). (A), (D) (2) (B), (D). (A), (C)
(3)(C),(A) (D), (B) (4)(B), (C), (D). (A)

47.Which one of the following is vastly absorbed by the ocean surface?


(1) Visible wave radiation (2) Medium wave radiation
(3) long wave radiation (4) Ultra-violet wave radiation

48.Who among the following gave the idea of the origin of monsoon in India?
(1) Al – Biruni (2) Al-Masudi
(3) Ibn – Hakkal (4) Ibn Khaldun

49.Arrange the following in order on the basis of year of meeting report


(A)COP21 ( B )KYOTO protocol
(C)First Earth SummiY (D)IPCC Report
Choose the correct answer from the options given below:
(1) (C), (B), (D). (A) (2)(B). (D). (A). (C)
(3)(A), (C). (D). (B) (4) (D). (C). (A). (B)

50. The term pen plane denotes to


(A)A level surface formed in the old age of a fluvial cycle
(B)A level surface formed by wind erosion
(C)A level surface formed by marine erosions
(D) A level surface formed by meeting of flood plain
(E )A level surface formed as an end product of erosion
Choose the correct answer from the options given below:
(1) (A) and (E) only (2) (B) and (C) only
(3) (B) and (D) only (4)(A) and (D) only

51.The concept of Lebensraum became popular in the discipline of political Geography after the publication
of –
(1) Making Political Geography (2) Geographical Pivot of History
(3)Cosmography (4))Politische Geographic

52.Which of the following atmospheric gases has not been represented by correct symbol?
(A) Neon – Ne ( B)Helium – He (C) Nitrous oxide - NO ( D)Krypton – KR
Choose the most appropriate answer from the options given below:
(1)(A) and (B) only (2)B) and (C) only
(3) (C) and (D) only (4)(A) and (D) only

34
PYQ 2021 BSSEI (M-8777279548) Coaching for NET Geography
53.Which one of the following group of states/UT recorded more than 35 percent of urban population
according to census 2011 data.
(1) Andhra Pradesh, Kerala, Chhattisgarh, Haryana
(2)Gujarat, Tamil Nadu, Kerala, Punjab
(3) Tripura, Kerala. Punjab. Chhattisgarh
(4)Sikkim, Kerala, Punjab, Goa

54.Which of the following describes central place model


(1) Spatial patterns of urban and out lying areas based on the flow of goods and services
(2) Spatial patterns of income groups
(3) Spatial patterns of residential places
(4) Spatial pattern of cultural places

55.Drift Theory of Wegener was postulated mainly to explain, which one of the following?
(A)Ice Age and Glacial movements
(B)Geological similarities of the coasts and Glaciations
(C)Distribution of landforms and ocean floors
(D) Major climatic changes in the world
Choose the most appropriate answer from the options given below:
(1)(A) and (B) only (2 ) (B) and (D) only
(3)(C) and (D) only (4)(A) and (D) only

56.Match List I with List II :


List I(Social Movement) List II(State of Origin
(A) Save Silent valley (1) Jharkhand
(B) Chipko (II) Gujarat
(C) Narmada Bachao (III) Uttarakhand
(D )Jungle Bachao (IV) Kerala
Choose the correct answer from the options given below:
(1)(A) - (1). B) - (II). (C) - (III). D) - (IV)
(2) (A)-(II). B) - (III). (C) - (IV). (D) - (1)
(3)(A) - (III). (B) - (IV), (C) - (I), (D) - (II)
(4)(A) - (IV), (B) - (III), (C) - (II), (D)

57.Hydraulic Slope Theory is given by


(1)C. Darwin (2) W.M. Davis
(3)W. Penck (4) R.E. Harton

58.Who among the following described geography as the science of map making?
(1) Thales of Miletus (2) Strabo
(3) Hecataeus (4) Cladius

59.Match List I with List II:


List I(Peaks) List II (Range)
(A) Gurusikhar (1) Nilgiri
(B) Dodabeta (II) Eastern Ghats
(C) Armakonda (III) Satpura
(D) Dhupgarh (IV) Aravalli
Choose the correct answer from the options given below:
(1) (A) – (II), (B) - (IV), (C) - (III), (D) - (1)
(2) (A) - (TV), (B) - (1). (C) - (II), (D) – (III)
(3) (A) – (II), (B) - (IV). (C) - (1), (D) - (III)
(4) (A) - (TV), (B) - (II), (C) - (1). D) - (III)

35
PYQ 2021 BSSEI (M-8777279548) Coaching for NET Geography
60.When a manufacturing firm locates very close to its raw material suppliers or to supplying finished
products, it is termed as
(1) Urbanisation economies (2) Internal economies
(3) Marketing economies (4) Localisation economies

61. Which of the following statements are correct about vertical distribution of temperature in oceans
(A)The sun ray's have no effect below 600 feet
(B)Inspite of movement of water; large part of sea is relatively cold
(C)80% of water in the oceans has a temperature permanently below 40°F
(D)The vertical rate of decrease in temperature from equator to pole is the same
Choose the most appropriate answer from the options given beloW
(1) (A). (B) and (C) (2) ( B ) ( CD) ( D) only
(3) ( C) (D ) Only (4 ) ( a) (d) Only

62.What will be the median of the distribution represented in the given graph?

1. 40 2.60 3.80 4.100

63.Which of the following are not the property/ies of correlation coefficient?


(A) r is independent of the choice of origin
(B ris independent of scale
(C) r lies between-1 and +1
(D = +1.0 indicates a perfectly negative correlation
(E)= -1.0 indicates a perfectly positive correlation
Choose the correct answer from the options given below:
(1) (A) and (B) only (2)(B) and (C) only
(3)(C) and (D) only (4)(D) and (E) only

64.Which one of the following refers to a relationship between a cultural group and natural environment?
(1) Cultural ecology (2)Cultural convergenc
(3)Cultural diffusion (4) Cultural hearth

65.Which of the following statements are correct about the state of Himachal Pradesh in India?
(A) High potentialities for hydro power generation
(B) High literacy
(c)High potentialities of horticulture development
(D) Rich in mineral resources
(E) High proportion of usable land
Choose the correct answer from the options given below:
(1) (A), B), (E) only (2)(A). (B), (D) only
(3)(C). D). (E) only (4)(A). (B). (C) only

36
PYQ 2021 BSSEI (M-8777279548) Coaching for NET Geography
66.The GIS based map can be
(A) Updated (B)Edited
(c) Displayed (D)Viewed in google web portal
Choose the most appropriate answer from the options given below:
(1) (A) and (B) only (2) (B). (C) and D) only
(3)(A), (B) and (C) only (4) (A). (B). (C) and (D) only

67. Match List I with List II:


List I Process List II Parameter
(A) Photosphere (1) Sunspot
(B Maunder minimum (II) Solar irradiance
(C) ABC (III) Palynology
(D) Pollen Grain (IV) Formation of layers of pollutants
Choose the correct answer from the options given below:
(1) (A) - (II). (B) - (III), (©) - (IV). (D) - (1)
(2) (A) – (II), (B) - (1). (C) - (IV), (D) - (III)
(3) (A) - (II), (B) - (IV), (C) - (1) (D) - (III)
(4) (A)-(II). (B) - (1). (C) - (III). (D) - (TV)

68.The correct sequence of the states in ascending order in terms of Mangrove vegetation coverage area is
(A) Goa (B)Andhra Pradesh
(C)Odisha (D)West Bengal
Choose the most appropriate answer from the options given below:
(1) (B). (A), (C). (D) (2)(A). (B). (C). (D)
(3)(B). (A). (D). (C) (4) (A), (B), (D), (C)

69.Which one of the following cluster regions in the world are great clusters of population?
(1) East Asia, North America, South Asia, Africa
(2) East Asia, South Asia, Europe, Central Africa
(3) East Asia, South Asia, Europe, North Africa
(4)East Asia, South Asia, Europe, North America

70.Which one of the following explains rapid growth of information and technology industry in South India
as compared to North India.
(A) Good Governance
(B)Good infrastructure and network accessibility
(C)Large number of higher technical institutes
(D)Lower Labour cost High proportion of manufacturing workers
Choose the correct answer from the options given below:
(1) (B). (C) and (E) only (2) (B). (C) and (D) only
(3)(A), (B) and (C) only (4) (A), (B) and (D) only

71.The range of pH value for unpolluted rain water is between


(1) pH4 and pH5 (2) pH5 and pH6
(3) pH6 and pH7 (4) pH7 and pH8

72.River regime with reference to flow refers to


(A) Pattern of seasonal flow of water
(B) Rivers differ in their regime due to climatic conditions
(C) River regime is represented mathematically by cumecs
(D) River regime of Narmada and Jhelum are similar
Choose the most appropriate answer from the options given below:
(1) (A) and (B) only (2) (B) and (C) only
(3) (C) and (D) only (4)(A) and (D) only

37
PYQ 2021 BSSEI (M-8777279548) Coaching for NET Geography
73.Which of the following food stuff consume highest amount of water
(1) Rice (2) Beef
(3) Chocolate (4) Banana

74.What is known as "Fossil water?


(1) Connate water (2) Meteoric water
(3)Plutonic water (4) Juvenil

75.The process of evaporation in ocean water is affected by


(A)salinity (B)latitude (C)season (D)winds

Choose the most appropriate answer from the options given below:
(1)(A) and (B) only (2) (B) and (C) only
(3)B). (C) and (D) only (4)(A), (B), (C) and (D)

76.Who among the following gave the concept of "Unity in Diversity"?


(1)A.V. Humboldt (2)Hettner
(3)C. Ritter (4)O. Paschel

77.Which of the following are not functional regione?


(A) Middle Ganga Plain (B) National Capital Region
(C) Malwa Plateau Region (D) Kolkata - Howrah Conurbation
Choose the most appropriate answer from the options given below:
(1) (A). (B) only (2)(A), (C) only
(3)(B),(C), D) only (4) (A) (B). (C) only

78.The outcome of Kyoto Protocol 1997 agreement was


(A)5.2% cut in emission of carbon dioxide
(B)Agreement can not be invalidated
(C)The protocol was to be effective from 1997
(D)The protocol was to be effective from 1999
Choose the most appropriate answer from the options given below:
(1)(A) and (B) only (2) (B) and (C) only
(3)(A) and (D) only (4)(C) and (D) only

79.Who among the following challenged the theory of origin of species?


(1) P.E. James (2) De Varies (3) H. Robinson (4) KK. Park

80.Who among the following challenged the theory of origin of species?


(1) P.E. James (2) De Varies
(3) H. Robinson (4) KK. Par

81.Given below are two statements : One is labelled as Assertion (A) and the other is labelled as Reason (R)
Assertion (A) :Fiord is not a suitable location for a port.
Reason (R): Fiords are drowned glaciated valleys with narrow deep channels along the coast.
In the light of the above statements, choose the most appropriate answer from the options given below:
(1) Both (A) and (R) are correct and (R) is the correct explanation of (A)
(2)Both (A) and (R) are correct but (R) is NOT the correct explanation of (A)
(3)(A) is correct but (R) is not correct
(4) (A) is not correct but (R

82.The term which describes that population will continue to grow even after fertility rates decline is known
as
(1) Demographic momentum (2) Demographic transition
(3) Rate of Natural increase (4) Rate of fertility decline

38
PYQ 2021 BSSEI (M-8777279548) Coaching for NET Geography
83.Which one of the following periods did archaeologists find the evidence of permanent human settlements?
(1) The paleolithic Period (2) The Bronze Age Period
(3) The Neolithic Period (4) The Classical Era Period

84.Identify the terms associated with ozone measurement


(A) Dobson (B)Spectrophotometer (C)Pyranometer
(D)Planimeter (E)Anemometer
Choose the correct answer from the options given below:
(1) (A) and (B) only (2)(B) and (C) only (3)(A) and (C) only (4)(D) and (E) only

85.Which of the following is not an appropriate strategy used to promote qualitative research validation?
(1) Peer review (2)Triangulation method
(3) Extended fieldwork (4)Random validation of assignments

86.Given below are two statements:


Statement I: The process in which one plate slides beneath another plate is zone of convergence.
Statement II: Sea floor spreading becomes more prominent along the zone of
convergences.
In the light of the above statements, choose the correct answer from the options given below:
(1)Both Statement I and Statement II are true
(2)Both Statement I and Statement II are false
(3)Statement I is true but Statement II is false
(4)Statement I is false but Statement II is true

87.Which of the following phenomena are not involved in global radiation balance?
(A)Pyranometer (B) Anemometer (C ) Albedo (D )Sensible heat
Choose the most appropriate answer from the options given below:
(1) (A) and (B) only (2) (B) and (C) only (3) (C) and (D) only (4)(A) and (D) only

88.Match List I with List II:


List I Rivers List II Types of Delta
(A) Missisippi (1) Arcuate
(B) Ganga (ID) Bird-foot
(C) Narmada (III) Cuspate
(D) Tiber (IV) Estuarine
Choose the correct answer from the options given below:
1.(A) - (1), B) - (III), (C) - (II), (D) - (IV) 2.(A) – (III), (B) - (II), (C) - (1). D) - (TV)
3.(A) – (II). (B) - 1). (C) - (IV). (D) – (III) 4.(A) - (TV). (B) - (II). (C) - (III). (D - (I)

89.Which one of the following states/U.T in India have recorded high decrease in infant mortality rate during
last 3 decades.
(1) Jammu and Kashmir (2) Uttar Pradesh (3)Tamil Nadu (4)Madhya Pradesh

90.Read the following passage carefully and answer the question given below
Climate change involves both internal and external factors. External one includes solar variability,
astronomical effects on the earth's orbit and volcanic activity whereas internal one includes natural variability
in the system, feedbacks within atmosphere, ocean and land surface. Recently, humaninduced climate change
on global and local scale has become a reality, primarily through changes in atmospheric composition and
surface properties. Climate changes involve continental drift, volcanic activity and solar output. Glacial-
interglacial cycles have been controlled by astronomical variations. The global temperature has increased by
0.5° C with greatest in the middle latitudes with warming particularly during the 1920 to 1940s and 1990s
with several warmest years. Long-term changes are ascribed to astronomical forcings. while short-term
changes linked to anthropogenic factors causing changes in atmospheric composition, including aerosol
loading, depletion of ozone, destruction of vegetation etc. Recently, climate change predictions with GCMS
are under taken for next 100 years depending on emission scenarios for green house gases and aerosols the

39
PYQ 2021 BSSEI (M-8777279548) Coaching for NET Geography
mean global temperature may increase in the range of 1.4 to 5.8°C, together with sea level rise. To conclude.
critical resarch data on cloud cover. radiation ocean processes atmospheric coupling and feedback processes
in the context of climate change are required.
Which one of the following rates of increase have been observed in global temperature in recent times?
(1) 1.0°C (2) 1.4°C (3) 0.5°C (4) 5.8°C

91. Which of the following conditions is essential for future research with respect to climate change?
(1) Hychological modelling (2) Runoff modelling
(3)Computer modelling (4)Atmospheric modelling

92. Which one of the following will be mean global temperature increase range till the year 2100?
(1)0.1-2.0°C (2) 1.5-4.8°C (3)1.4-5.8°C (4)2.0-3.0°C

93. Which of the following activities do not accentuate short-term changes in atmospheric composition?
(1) Aerosol loading (2) Astronomical forcing (3) Loss of flora (4) Depletion of

94. Which of the following elements is not associated with climate change?
(1) Solar variability (2) Upwelling interactions
(3)Volcanic eruptions (4) Astronomical periodicities

95.Read the following passage carefully and answer the question given below
Population Geography, a sub field of Geography, studies spatial variations in distribution composition, migration and
growth of population to the terrain, in a way it involves demography in geographic perspective. If focuses on questions
such as where populations are found and how the size and composition of these populations in regulated by demographic
processes of fertility, mortality and migration contributions to population geography are cross-disciplinary. Disciplines
other than geography contributing to population geography include demography, sociology and economics. Since its
inception, population geography has taken at least three distinct forms. The earliest emerging in the 1950s, focused on
the systematic study of population distribution as a whole and spatial variations in dynamic characteristics of population.
Trewartha, Zelin sky. W.A.V Clark and other from U.S.A. and J.B Garnier and Pierre George in France Pioneered this
phase. Population geography has widered its scope to cover different variety of themes with time. It studies various
population characteristics including gender, religion age, disability racrete by grouping population characteristics into
political and administrative units. All such variables go far beyond the vital statistics of births. deaths and marriage
/migrations. Geographers have studied relations between demographic growth. displacement and access to resources at
the International scale. Examples are density maps prepared using choropleth, isoline & dots
Who among the following is not a U.S. Geographer pioneering population geography as a part of
spatial science?
(1) G.T. Trewartha (2) Pierre George (3)Wilbur Zelinsky (4) W.A.V. Clark

96. In which of the following decades, Population Geography greatly focused on the systematic study of
distributions and spatial differentiations in various attributes of population characteristics
(1) 1960s (2) 1960s (3) 1980s (4) 1990s

97. Which one of the following group of disciplines, other than geography, has contributed significantly to
studies in population geography?
(1) Psychology. Anthropology and History (2) Anthropology, Statistics and Sociology
(3)Sociology, Demography and Economics (4) Economics, Sociology and History

98. Which one of the following vital statistics is related with demographic characteristics of population studied
in Population Geography?
(1) Age and sex (2)Religious composition and race
(3) Birth, deaths, marriage/migration (4) Literacy and workforce

99. Which one of the following population characteristics does not belong to vital statistics?
(1) Births (2) Deaths
(3) Age and sex (4) Marriage /migration

40
PYQ 2022 BSSEI (M-8777279548) Coaching for NET Geography

Nta NET Geography July 2022


1. The geometrical shape formed by a group of building is termed as :
1. Site 2. Pattern 3. Location 4. Situation

2. Which of the following pairs are correctly matched ?


A. Carl Ritter - Continentality concepts
B. Alexander Von Humboldt - Teleological Approach
C. Oscar Peschel - Das Ausland
D. Varenius - Critique of pure Reason
E. Immanuel kant - General History of Nature of the Heavens
Choose the correct answer from the options given below:
1. A and C only 2. B and E only
3. D and E only 4. C and E only

3. Which one of the following is the boundary line between India and Pakistan?
1. Durand Line 2. Blue Line 3. Radcliffe Line 4. McMohan Line

4. Arrange top five wheat producing states of India ( in 2020 – 21) in descending order.
A. Uttar Pradesh B. Haryana C. Rajasthan
D. Punjab E. Madhya Pradesh
Choose the correct answer from the options given below:
1. A , D , B , E , C 2. A, D, E , B, C
3. A, E , B , D , C 4. D, A , B, E, C

5. Arrange the following continents of the world in descending order on the basis of current population density
( persons per km2):
A. Asia B. North America C. Africa D. Europe
Choose the correct answer the options given below:
1. C, B, A , D 2. A, C, D, B 3. B, A, C, D 4.D, A, B, C

6. How does Gunner Mrydal ( 1957) describe the generation of regional inequalities at the national and
international levels in his theory of Circular Cumulative Causation?
1. Forward and backward linkages 2. Push and pull factors
3. Backwash and spread effects 4. Balance and unbalanced growth

7. Given below are two statements : One is labeled as Assertion A and the other is labeled as Reason R
Assertion A: Any lack of water to satisfy the normal needs of agriculture. Livestock industry or human
population may be termed as a drought
Reasons R: Drought is distress situation caused by lack of rainfall
In the light of the above statements, choose the most appropriate answer from the options given below:
1. Both A and R are correct and R is the correct explanation of A
2. Both A and R are correct but R is NOT the correct explanation of A
3. A is correct but R is not correct
4. A is not correct but R is correct

8. Given below are two statements: one is labeled as Assertion A and the other is labeled as Reason R
Assertion A : In terrestrial ecosystems ( grassland , forests ) the pyramid of biomass is upright with broad base
Reasons R : In terrestrial ecosystems the biomass increases from primary to tertiary consumers
In the light of the above statements, choose the most appropriate answer from the options given below
1. Both A and R are correct and R is the correct explanation of A
2. Both A and B are correct but R is NOT the correct explanation of A
3. A is correct but R is not correct
4. A is not correct but R is correct

41
PYQ 2022 BSSEI (M-8777279548) Coaching for NET Geography
9. Proper sequence of Wilson cycle is :
A. Continental rupture and formation of wide ocean basin with passive margins
B. Formation of Island arcs and arc – continent orogen
C. Oceanic lithosphere fractures and new subduction boundaries take shape
D. Plate reverse their motion, and the ocean basin to close
E. Formation of continental suture
1. A , C, D, B , E 2. A, D, C, B, E
3. A, D, C, E, B 4. A, C, E, D, B

10. Given below are two statements:


Statement I: Shafi calculated Agricultural productivity through ranking co – efficient method in the year 1990
Statement II: Agricultural productivity as an input- output ratio was calculated by khusro in the year 1964.
In the light of the above statements, choose the correct answer from the options given below
1. Both Statement I and statement II are true
2. Both statement I and Statement II are false
3. Statement I is true but Statement II is false
4. Statement I is false but Statement II is true

11. Identify the type of universe origin as discussed in the cosmology of vedas.
A. Philosophical origin
B. Artistic origin
C. Instrumental origin
D. Chemical origin
Choose the correct answer from the options given below:
1. A and C only 2. B and D only
3. A , B and C only 4. C and D only

12. Match List I with List II


List I ( Scale system) List II ( Data type measured)
A. Nominal I. Altitude from mean sea level
B. Ordinal II. Soil type
C. Interval III. Monthly income ( in Rs.)
D. Ratio IV. Town size
Choose the correct answer from the options given below:
1. A – II, B – IV , C – I , D – III 2. A – III, B – II, C – I, D – IV
3. A – III,B – IV, C – I, D – II 4. A – II, B – III, C – I ,D – IV

13. Which of the following statement is not correct?


A. More than one – half of the global population reside in areas below 200 meters and above mean sea level
B. More than 50 % of the global population lives on 25% of the land surface
C. 90% of the global population reside in the northern hemisphere
D. The margins of continents are highly populated than interiors
Choose the correct answer from the options given below:
1. C only 2. B only 3. D only 4. A only

14. Ammonification is an important process in :


1. Phosphorus 2. Carbon cycle
3. Nitrogen cycle 4. Oxygen cycle

15. Which ministry of government of India looks after Census operations in India?
1. Ministry of Statistics and programme Implementation
2. Ministry of Education
3. Ministry of Home Affairs
4. Ministry of Finance

42
PYQ 2022 BSSEI (M-8777279548) Coaching for NET Geography
16. What is debt Service Ratio?
1. It is the proportion of a country’s export earning that it needs to use to meet its debt
repayment
2. It is the proportion of a country’s total national income that it needs to use to meet its debt repayment
3. It is the proportion of a country’s income from service sector that it needs to use to meet its debt
repayment
4. It is the proportion of a country’s total income from manufacturing sector that it needs to use to meet
its debt repayment

17. Which one of the following model of the internal structure of the city emphasizes the importance of
transport routes?
1. Concentric model 2. Sector model
3. Multiple nuclei model 4. Urban social area model

18. To examine the state of world environment and development beyond 2000. UN General Assembly set up
World Commission on Environment and Development ( WCED) in 1983. Which is also known as:
1. Commission on Sustainable Development
2. Brundtland Commission
3. Agenda
4. Chipko Commission

19.Which of the following are Characterized by stratovolcanose?


A. Basaltic magma
B. Involves inter layered strata
C. Magma is gassy and highly viscous
D. Magma emerges under low pressure
Choose the correct answer from the options given below:
1. A and D only 2. B and D only
3. A and C only 4. B and C only

20. The report entitled “ our common Future” was submitted in 1987 by :
1. Food and Agricultural Organization
2. World Meteorological Organization
3. Intergovernmental Panel on climate change
4. Brundtland Commission

21. Match List I with List II:


List I ( Minerals / Power resources) List II ( Name of mines)
A. Manganese I. Dungarpur. Malanjkhand
B. Copper II. Balaghat , Chhindwada
C. Bauxite III. North Karnpur. Jhilmili
D. Coal IV. Bolangir , Udgeri
Choose the correct answer from the options given below:
1. A – II, B – I , C – IV, D – III
2. A – II, B – I, C – III, D – IV
3. A – I, B – II, C – IV, D – III
4. A- III, B – I, C – IV, D – II
∑𝑥 2
22. Crop Diversification Index = (∑𝑥)2
In the above formula, ‘x’ refers to:
1. Gross cropped area
2. Total cropped area occupied by each crop
3. Net sown area
4. Net sown area occupied by each crop

43
PYQ 2022 BSSEI (M-8777279548) Coaching for NET Geography
23. Which of the following countries have adopted pro- natal population policy?
A. Sri Lanka B. Spain C. Malaysia
D. Myanmar E. Japan
Choose the correct answer from the options given below:
1. A only 2. A, C and D only
3. C , D and E only 4. C and E only

24. Which of the following are defining features of positivism?


A. Metaphysics B. Empiricism
C. Normative values D. Unification of Scientific laws
Choose the correct answer from the options given below:
1. B and D only 2. A and B only
3. A and C only 4. C and D only

25. Which one of the following fault has greater risk of landslides?
1. Strike – Slip or Transcurrent fault
2. Normal fault
3. Reverse fault
4. Over thrust fault

26. Which of the following statements are incorrect about Weber’s theory of Industrial Location?
A. To determine where the location of industry lie. Weber calculated the ratio between the weight of
localised material and the weight of the product
B. He designated this as the Material Index
C. If the index is less than one (1) the location is oriented towards the consumption centre
D. If the index is more than one (1), the location is material – oriented
E. if the index is more than one (1), the location is oriented towards the consumption centre
Choose the correct answer from the options given below:
1. B and E only 2. C and E only
3. D and E only 4. E only

27. Which of the following statements is/ are correct?


A. The BOD (Biological Oxygen Demand) of sewage water is very high
B. The aerobic organisms rapidly increase in sewage water due to low content of dissolved oxygen
C. Ecology is the study of interrelationship between the organism and their physical environment
D. parasites are those organisms, which do not depend on other living organisms for their food
Choose the correct answer from the options given below:
1. A only 2. C only 3. A and C only 4. B and D only

28. The dating and study of annual rings of trees is called:


1. Morphology 2. Eccentricity
3. Precession 4. Dendrochronology

29. How is ‘Cultural Hearth’ defined?


1. It is concerned with the power of spreading of cultural phenomena on the earth from a source or
sources of origin
2.The birth place of a cultural group associated with a particular cultural landscape
3. Cultural parameter that specify learned behaviour
4. Cultural regions showing cultural complexes and landscapes

30 Arrange the following in the correct sequence of detritus food chain described by Odum (1970) based on
mangrove leaves:
A. Detritus B. Small Fish C. Mangrove leaves
D. Large Fish E. Crabs and Shrimps

44
PYQ 2022 BSSEI (M-8777279548) Coaching for NET Geography
Choose the correct answer from the options given below:
1. C, B , A , E , D 2. C , E, B , A , D
3. C, E, A , B, D 4, C, A, E, B , D

31. Knock – knee syndrome (outward bending of legs from knees) is mainly caused due to:
1. Fluoride pollution 2. Lead pollution
3. Mercury pollution 4. Carbon – di – oxide pollution

32. Which one of the following is not a Particularly Vulnerable Tribal Group ( PVTGs)?
1. Vantangia 2. Marram Nagas 3. Seharias 4. Reangs

33. Match List I with List II:


List I ( Scholar) List II ( Concept/ Term / Contribution)
A. Gaylord Nelson I. Ecology
B. A. E. Douglas II. Ecological Niche
C. Charles Elton III. Earth day
D. Ernst Haeckel IV. Dendrochronology
Choose the correct answer from the options given below:
1. A – I , B – III, C – IV, D – II
2. A – III, B – IV, C – II, D – I
3. A – II, B- III, C – I, D – IV
4. A – IV, B – I, C – III, D – II

34. Who has authored the famous book, ‘Muqaddimah’?


1. Ibn Batuta 2. Al Biruni 3. Al – Idrisi 4. Ibn- Khaldun

35. Which perspective in geography is associated with Ferdinand de Saussure?


1. Positivism 2. Behaviouralism 3. Feminism 4. Structuralism

36. Match List I with List II:


List I ( Thematic Maps) List II ( Data Type represented: Example)
A. Qualitative I. Interval and Ratio Data: Chropleth maps
B. Semi - Quantitative II. Virtual maps generated digitally using computers:
Landform maps
C. Quantitative III. Nominal Data: Chorochromatic maps
D. Special Symbolization/Ordinal data: Choroschematic map
Choose the correct answer from the options given below:
1. A – III, B – I, C – II, D – IV
2. A – III, B – IV, C – II, D – I
3. A – III, B – IV, C – I, D – II
4. A – III, B – II, C – IV, D - I

37. Five Indian metros with respect to total population are arranged in descending order, which of the
following sequence is correct?
1. Mumbai, Delhi, Kolkata, Chennai, Bengaluru
2. Delhi, Kolkata, Mumbai, Chennai, Bengaluru
3. Mumbai, Kolkata, Delhi, Bengaluru, Chennai
4. Mumbai , Delhi, Chennai, Kolkata , Bengaluru

38. Which of the following are cumulative atmospheric disasters?


A. Tornado disaster B. Flood disaster C. tsunami disaster
D. Heat wave disaster E. Landslide disaster
Choose the correct answer from the options given below:
1. A and B only 2. B and C only
3. A, B and D only 4. B and E only

45
PYQ 2022 BSSEI (M-8777279548) Coaching for NET Geography
39. According to Census of India. 2011 which are the top- three states in descending order with the largest
size of Scheduled Tribe (ST) population?
A. Madhya Pradesh, Odisha, Maharashtra
B. Madhya Pradesh, Maharashtra, Odisha
C. Madhya Pradesh, Odisha, Rajasthan
D. Madhya Pradesh, Rajasthan, Odisha
Choose the correct answer from the options given below:
1. A only 2. B only 3. C only 3. C only 4.D only

40.Which of the following is associated with international agreement on ‘substance that deplete the ozone
layer’
1. Paris Agreement 2. Marrakech Accord
3. Kyoto Protocol 4. Montreal Protocol

41.Match List I with List II:


List I ( Name of the Scholar) List II ( Principle/ Law/ Model)
A. Sadler I. Net impact of population growth on economy
B. Ricardo II. Intervening opportunity model
C. Thompson III.Fertility varies inversely with density
D. Stouffer IV. Demographic Transition model
Choose the correct answer from the options given below:
1. A – I, B – II, C – III, D – IV
2. A –II, B – III, C –IV, D- I
3. A – III, B – I, C – IV, D – II
4. A –IV, B –II, C – I, D – III

42. Two of the following ocean currents are responsible for reducing the temperature of bordering coastal
zones:
A. Benguela current B. Kurosiwo current
C. Brazil current D. Canaries current E. Gulf Stream
Choose the correct answer from the options given below:
1. A and B only 2. A and C only
3. A and D only 4. D and E only

43.Which two of the following alphabetical symbols represent ‘the wet tropics’ climate according to Koppen’s
climate classification?
A. BWh B. Af C. Dfe D. Am E. BSh
Choose the correct answer from the options given below:
1. A and E only 2. B and D only
3. B and C only 4. A and C only

44. Given below are two statements : one is labeled as Assertion A and the other is labeled as Reason R
Assertion(A): During an equinox the length of daylight is 12 hours everywhere on Earth
Reasons (R) : On equinox the circle of illumination passes directly through the poles dividing the latitudes in
half
In the light of the above statements, choose the most appropriate answer from the options given below
1. Both A and R are correct and R is the correct explanation of A
2. Both A and R are correct but R is NOT the correct explanation of A
3. A is correct but R is not correct
4. A is not correct but R is correct

45. Which term is used by W. Penck to denote waning development of landforms?


1. Primarumpf 2. Aufsteigende entwickelung
3. Gleichformige entwickelung 4. Absteigende entwickelung

46
PYQ 2022 BSSEI (M-8777279548) Coaching for NET Geography
46. Match List I with List II:
List I ( Places of Earthquekes) List II ( Tectonic Environment)
A. Bam ( Iran) I. Subduction zone
B. Mexico city II. Transform boundaries
C. San Francisco III. Under sea subduction
D. Java IV. Blind faults
Choose the correct answer from the options given below:
1.A - IV, B – I, C – III, D – II
2. A – IV,B – I, C – II, D – III
3. A – I, B – IV, C - II, D – III
4. A – I, B – IV,C – III, D – II

47. Which of the following is/are not the cause/ cause of floods in Brahmaputra river system?
A. Tributaries of Brahmaputra bring huge quantity of water and silt, heavy deposit of silt has raised the river
bed
B. Faulty agricultural practices
C. Occasional earthquakes have brought about changes in the course of river
D. Broadness of the Brahmaputra Valley
E. Very high population pressure has forced the population to live in the flood prone area
Choose the correct answer from the options given below:
1. A only 2. B and C only
3. B and D only 4. E only

48. Match List I with List II:


List I ( wind Type) List II ( Wind Charecteristics)
A. Bores I. Warm and gentle
B. Eurus II. Advancing storm
C. Notus III. Strong and cool with clear skies
D. Zephyrus IV. Dreaded and balmy
Choose the correct answer from the options given below:
1. A – II, B – I, C – III, D – IV
2. A – III, B – I, C – II, D – IV
3. A – IV, B – II, C –I, D –III
4. A – I, B – IV, C – II, D – III

49. Which of the following pairs match correctly?


A. Wall map - 1: 1,980
B. Large scale plan - 1:100
C. Topographical map -1: 250.000
D. Atlas map - 1: 250
E. Cadestral map -1: 3,960
Choose the correct answer from the options given below:
1. A and E only 2. A and C only
3. B, C and E only 4. B and D only

50. Arrange the following trace gases in correct sequence of their concentration in atmosphere in descending
order:
A. Methane B. Krypton C. Helium
D. Neon E. Hydrogen
Choose the correct answer from the options given below:
1. E, B, D , A , C 2. D, B, E, A , C
3.D,C, A, B, E 4. D, E, B, A, C

47
PYQ 2022 BSSEI (M-8777279548) Coaching for NET Geography
51. Match List I with List II:
List II List II
A. Von Thunen I. Central Place Theory
B. August Losch II. Agricultural Location
C. Doi III. Theory of Industrial Location
D. Christaller IV. Crop combination
Choose the correct answer from the options given below:
1. A – II, B – IV, C – III, D – I
2. A – II, B – I, C – IV, D – III
3. A – II, B – III, C – IV, D – I
4. A – I, B – II, C – III, D – IV

52. Which of the following statements are True?


A. Estimates for Life expectancy can be obtained from Life table
B. Improvement in Life expectancy is one of the causes of population ageing
C. Infant Mortality Rate is defined as the number of deaths of children below one year per 1000 mid
year population
D. Malthus proposed demographic transition theory
Choose the correct answer from the options given below:
1. A and B only 2. C only 3. C and D only 4. A, B and C only

53. Identify two Swedish scholar who contributed in the field of climatology
A. Gabriel Daniel Fahrenheit B. Anders Celsius
C. Vilhelm Bjerknes D. Tor Bergeron E. Gaspard Gustav Coriolis
Choose the correct answer from the options given below:
1. A and E only 2. B and C only
3. B and D only 4. B and E only

54. Arrange the following left bank tributaries of The Ganga from west to east
A. Ram Ganga B. Gomti C. Gandak D. Ghaghra
Choose the correct answer from the options given below:
1. B , D , C and A only 2. A , C ,D, and B only
3. A , B , D and C only 4. A , C ,B and D only

55. Identify the correct statement from the following:


1. Wind is deflected to its right in the southern hemisphere
2. Geostrophic wind blows parallel to the isobars
3. Wind speed increases with friction
4. Wind speed is inversely proportional to pressure gradient force

56. Cambay Graben is associated with which one of the following earthquake?
1. Bhuj earthquake 2. Latur earthquake
3. Jabalpur earthquake 4. Quetta, Baluchistan earthquake

57. Who is the first person to coin the term ‘Geopolitik’?


1. Friedrich Ratzel 2. Rudolf Kjellen
3. Karl Haushofer 4. Peter Kropotin

58.Which one of the following is not correctly matched?


1. Cherry Blossoms – Karnataka
2. Norwesters – West Bengal, Assam and Odisha
3. Loo – North India
4. Mango Showers – Maharashtra

48
PYQ 2022 BSSEI (M-8777279548) Coaching for NET Geography
59. In the expansion of tourism industry. Which events played a significant role in a chronological order
A. Advent of wheel B. End of World War II
C. Thomas Cook arranged the first package tour between Leicester and Loughborough in U.K.
D. Usage of Jet planes for movement of passengers
Choose the correct answer from the options given below:
1. A, C , C and D only 2. A, B, C and D only
3. A,D, B and C only 4. A , D , C and B only

60. The elements – sits, situation and association fall in which order of Aerial image interpretation?
1. Primary 2.Secondary 3. Tertiary 4. Higher
61. A regressive population age structure is likely to be encountered in a country:
1. Which has completed the demographic transition
2. Where the young and old population are balanced
3.Which has young population
4. Which is in the early expanding stage of demographic transition

62. Which are the three component indicators used by M.D. Morris (1979) to calculate the physical Quality
of Life Index ( PQLI)?
1. Infant Morality Rate, Life Expectancy at age One, Basic Literacy at age 15
2. Maternal Mortality Rate. Literacy at age 7 years. Life Expectancy
3. Neo – natal Mortality Rate. Women Literacy Rate. Life Expectancy at age one
4. Infant Mortality Rate. Out of School Children. Life Expectancy at age one

63. Which of the following oil field / fields of India is / are correctly matched?
A. Assam - Moran – Hugrijan oil field
B. Gujrat - Lunej, Kalol oil fields
C. East Coast offshore oil fields - Olkad , Asjol
D. West Coast offshore oil field - Bassein ,Aliabet
Choose the correct answer from the options given below:
1. A and B only 2. A , B and D only
3. A, B and C only 4. C and D only

64. Given below are two statements:


Statement I: Digital Elevation Model ( DEM) is a raster format
Statement II: All pixels are of different shape and size in DEM
In the light of the above statements , choose the correct answer from the options given below
1. Both Statement I and Statement II are true
2. Both Statement I and Statement II are false
3. Statement I is true but Statement II is false
4. Statement I is false but Statement II is true

65. In 2020- 21. India exported poultry products to which of the following countries?
A. Oman B. Maldives C. Pakistan D. Indonesia E. Thailand
Choose the correct answer from the options given below:
1. A only 2. A and B only 3. C and E only 4.A , B and D only

66. An ecological succession which begins on a place rich in inorganic contents but poor in organic contents
is called:
1. Autogenic succession 2. Allogenic succession
3. Induced succession 4. Autotrophic succession

67. Which one of the following is not correctly matched?


A. devprayag - Bhagirathi + Alaknanda B. Rudraprayag - Mandakini + Alaknanda
C. Karnprayag - Bhagirathi + Pindar D. Vishnuprayag – Dhauliganga + Alaknanda

49
PYQ 2022 BSSEI (M-8777279548) Coaching for NET Geography
68. Which one of the following local wind is a Katabatic wind ?
1. Chinook 2. Foehn 3. Santa Ana 4. Mistral

69. Which of the following are methods of linear contrast image enhancement?
A. Minimum – maximum stretch
B. Histogram equalization
C. Density slicing
D. Histogram normalization
E. Saturation stretch
Choose the correct answer from the options given below:
1. A and B only 2. A and C only
3. B and D only 4. A and E only

70. Niche Market tour operators have increased in number to satisfy specialist holiday. Which includes:
A. Theme parks B. Cruise
C. Sports tourism D. Family Gathering tourism
Choose the correct answer from the options given below:
1. A , B and C only 2. A , B and D only
3. A, C and D only 4. D only

71. Map is showing the passes of Kashmir Himalayas. Which of the sequence of labelled passes is ascending
order is correct?

A. 1. Karakoram 2. Quara Tag 3. Lanak La 4. Chang La 5. Umasi La 6. Banihal 7. Pir Panjal


8. Zoji La
B. 1. Karakoram 2. Quara Tag 3. Chang La 4. Lanak La 5. Umasi La 6. Banihal 7. . Pir Panjal 8.
Zoji La
C.1. Karakoram 2. Aghil 3. Lanak La 4. Umasi La 5. Chang La 6.Banihal 7. Pir Panjal 8. Zoji La
D. 1. Karakoram 2. Saser La 3. Lanak La 4. . Chang La 5. Umasi La 6. Pir Panjal 7. Qura Tag 8. Zoji
La

72. Which one of ther following term is not related to Continental Drift Theory?
1. Panthalasa 2. Paleoclimate
3. Sea floor spreading 4.Jig saw fit

50
PYQ 2022 BSSEI (M-8777279548) Coaching for NET Geography
73. Which agency is associated with IKONOS remote sensing data?
1. Digital Globe 2. Imagesat International Inc.
3. Digital Image 4. ORB Image

74. Who among the following authors have proposed ‘Tourism Area Life Cycle (TALC)’model illustrating
how tourism develops and changes over time in a region?
1. John Glasson. 1956 2. Richard Butler .1980
3. David Harvey . 1945 4. Robert Prosser . 1982

75. As per the Nearest Neighbourhood analysis , an index value of 2.5 suggests that the settlements are:
1.Clustered 2. Highly Clustered 3. Randomly distributed 4. Evenly spaced

76. Which of the following statement is / are correct?


A. Denudation involves weathering , mass wasting and crosion
B. Physical Weathering and Mechanical Weathering process differ only in scale
C. In dry climates, salt – crystal growth and weathering lead to the formation of distinctive hollowed
– out near the bases of sandstone cliff
D. Exfoliation occurs when temperature changes cause minerals to expand and contract at different
rates
Choose the correct answer from the options given below:
1. B only 2. A and C only 3. B and D only 4. A only

77. What is true about the Heartland theory?


A. It was published thrice in 1904 ,1914 and 1943
B. It was put forward by a German geographer HJ Mackinder
C. It was based on land versus sea power struggle in world History
D. The predictions of the theory came to be true and Germany became the superpower in the world
Choose the correct answer from the options given below:
1. A only 2. A and C only 3. C only 4. C and D only

78.Which one of the following stage in demographic transition model describes the situation of high and stable
birth rate along with high and fluctuating death rate?
1. Early expanding stage 2. Late expanding stage
3.High stationary stage 4. Low stationary state

79. The proper sequence of ridges from South to North in the Indian Ocean is
A. Kergelen – Gausserberg ridge: Chagos – Saint Paul ridge: Socotra – Chagos ridge: Laccadive – Chagos
ridge
B. Kergelen – Gausserberg ridge: Chagos – Saint Paul ridge: Socotra – Chagos ridge: Andaman – Nicobar
ridge
C. Chagos – Saint Paul ridge: Kergelen – Gausserberg ridge: Socotra – chagos ridge : Laccadive – Chagos
ridge
D. Chagos – Saint Paul ridge: Kergelen – Gausserberg ridge: Socotra – chagos ridge: Andaman – Nicobar
ridge
Choose the correct answer from the options given below:
A. A and D only B. A , B and C only C. A and B only D. B , C and D only

80. Match List I with List II:


List I ( Iron and Steel Plants) List II ( Source of Iron ore)
A. TISCO , Jamshedpur I. Kemmangundi mines
B. VISL ,Bhadravati II. Noamundi Gurumahisani mines
C. HSL Bhilai III. Kiriburu mines
D. BSL Bokaro IV. Dalli – Rajhara range

51
PYQ 2022 BSSEI (M-8777279548) Coaching for NET Geography
Choose the correct answer from the options given below:
1. A – II, B – III, C – IV, D – I 2. A – I, B – III, C – IV, D – II
3. A – II, B - I, C – IV, D – III 4. A – I, B –IV, C – II, D – III

81. As per Census of India , 2011 arrange the following Union Territories in a descending order according to
the recorded literacy rates.
A. Chandigarh , Puducherry , Daman & Diu, Dadar Nagar Haveli
B. Puducherry ,Chandigarh , Dadar Nagar Haveli Daman & Diu
C. Chandigarh , Dadar Nagar Haveli , Puducherry , Daman & Diu
D.Daman & Diu, Puducherry , Chandigarh , Dadar Nagar Haveli
E. Daman & Due , Chandigarh, Puducherry, Dadar Nagas Haveli
Choose the correct answer from the options given below:
1. A only 2. A and E only 3. A , B and C only 4. D only

82. In which one of the following fronts , warm air is completely displaced from the ground surface?
1. Occluded front 2. Cold front 3. Warm front 4. Stationary front

83. Which one of the following statistical technique is used to analyse the interdependence of the variables?
1. Location Quotient 2. Standard Score
3.Co – efficient of variability 4.Principal Components Analysis
84. Match List I with List II:
List I ( Transport Network Measures and List II ( Definition)
Connectivity)
A. Eta Index I. Measures the level of connectivity in a graph and
is expressed by the relationship between the number
of links over the number of nodes
B.Theta Index B. Average length per link
C. Beta Index C. Average amount of traffic per intersection
D. Alpha Index D.It evaluates the number of cycles in a graph in
comparison with the maximum number of cycles
Choose the correct answer from the options given below:
1. A – II, B – I , C – III, D – IV 2. A – II, B – IV, C – III, D – I
3. A – II, B – III, C – IV, D – I 4. A – II, B – III, C – I, D – IV

85. In India National Ambient Air Quality Monitoring (NAAQM) programme was initiated by:
1. National Environmental Research Institute 2. Central Pollution Control Board
3. India Meteorological Department 4. National Institute of Urban Affairs

86. Which of the following statements are true?


A. A country achieves replacement level of fertility when it attains a Total Fertility Rate of 2.1
B. Fecundity refers to the capacity of women to bear children
C. According to Malthus, war is a preventive check
D. Crude death rate refers to the number of deaths per 1000 mid year population
Choose the correct answer from the options given below:
1. A , C and D only 2. B and C only 3. A , B and C only 4. B , D and A only

87. Which of the following is correct about the Raster data model in GIS?
A. Raster data model is a simple data structure B. All pixels are of same size and shape
C. Smaller file size D. Raster data model is compatible with remote sensing imageries
E. Difficult overlay operations
Choose the correct answer from the options given below:
1. A , B and D only 2. A and C only 3.C and E only 4. A , B and E only

88. A point data is :


1. 0D – Dimensionless 2. 1D – 1 dimensional 3. 2D – 2 dimensional 4. 3D – 3 dimensional

52
PYQ 2022 BSSEI (M-8777279548) Coaching for NET Geography
89. A scholar of which origin coined the term ‘ permafrost’ to explain the frozen characteristics of the Siberian
soils?
1.American 2. British 3.French 4. German

90. Match List I with List II:


List I ( Name of soil) List II (Area)
A. Calcareous clayey soil I. Chotanagpur highlands
B. Red loamy soil II. Mid- altitudinal zones of Himalaya
C. Podzolic soil III. Desert soil
D. Soil with low organic matter. Low nitrogen and IV. Khadar
other plant nutrients
Choose the correct answer from the options given below:
1. A – IV, B – I, C – II, D – III 2. A – IV, B – II, C – I, D – III
3. A- IV, B – II, C – III, D – I 4. A – II,B – I, C –IV,D – III

91. Read the following passage(91-95) carefully and answer questions:


Movements of people, goods and information have always been fundamental components of human societies.
Contemporary economic processes have been accompanied by a significant increase in mobility and
accessibility. Although this trend can be traced back to the industrial revolution. It significantly accelerated
in the second half of the twentieth century as trade was liberalized , economic blocs emerged and the
comparative advantages of global labor and resources were used more efficiently, Societies have become
increasingly dependent on their transport systems to support a wide variety of activities ranging , among
others, from commuting , supplying energy needs, to distributing parts between manufacturing facilities and
distribution centers, The unique purpose of transportation is to overcome space , which is shaped by a variety
of human and physical constraints such as distance, time , administrative divisions and topography . Jointly ,
they confer a friction to any movement . Commonly known as the friction of space . There would be no
transportation without geography and there would be no geography without transportation. The goal of
transportation is thus to transform the geographical attributes of freight , people or information, from an origin
to a destination , conferring on them an added value in the process. The convenience at which this can be done
– transportability – varies considerably .transportability refers to the ease of movement of passengers, freight
or information.It is related to transport costa as well as to the attributes of what is being transported ( fragility
. perishable. Price) . political factors can also influence transportability such as laws. Regulation , borders and
tariffs. The specific purpose of transportation is to fulfill a demand for mobility . Since transportation can only
exist if it moves people , freight and information around . Otherwise it has no purpose. This is because
transportation is dominantly the outcome of a derived demand.
Choose the correct answer from the options given below:
1. A and B only 2. A , B and D only
3.A , B and C only 4. C and D only

92. Societies have become increasingly dependent on their transport systems for which of the following
reasons?
A. Commuting B. Supplying energy needs
C. For manufacturing and distribution D. All of the above
Choose the correct answer from the options given below:
1. D only 2. A and B only 3. B and C only 4. A and C only

93. Given below are two statements:


Statement I: The specific purpose of transportation is to fulfill a demand for mobility. Since transportation can
only exist if it moves people . Freight and information .
Statement II: Transportation is dominantly the outcome of a derived demand .
In the light of the above statements , choose the correct answer from the options given below
1. Both Statement I and Statement II are true 2. Both Statement I and Statement II are false
3. Statement I is true but Statement II is false 4. Statement I is false but Statement II is true

53
PYQ 2022 BSSEI (M-8777279548) Coaching for NET Geography
94. What is ‘transportability’?
1. The ease of movement of passengers. freight or information.
2. The difficulty of movement of passengers. Freight or information.
3. The outcome of movement of passengers. freight or information.
4. The laws and regulations governing the movement of passengers.

95. What is understood by the term ‘friction of space’?


1. Human and physical constraints that creates a friction to any movement of people. Freight
and information.
2. Human constraints that create a friction to any movement of people . freight and information.
3. Physical or natural constraints that create a friction to any movement of people. freight and
information.
4.Human and physical constraints that create a friction to any movement of information only.

96. The Greek scholar Eratosthenes. Who lived in the third century B. C.. was the first to use the word
geography. But humans were investigating geographic questions long before this. The Greek geographers
credit the beginning of geographical writings to Homer. But the earliest known map was made by the
Sumerians in about 2700 B. C. The study of geography goes back to the beginning of human scholarship. The
first amazing period of intellectual ferment that is part of the written tradition of the Western world took
place in ancient Greece. Culminating in the fourth and third centuries B. C . The Greeks developed the
procedures we describe as the scientific method, Plato. Who developed the deduction procedures is the most
often quoted buy those who prefer to give theory the position of chief importance. Aristotle, who developed
the inductive procedures, preferred to formulate his concepts as generalizations of observed facts. Aristotle
insisted on the importance of direct observation . instead of making logical deduction from theory. Among the
ancient Greek Philosophers. Two basic traditions of geographic study are to be found. One is the mathematical
tradition. Starting with Thales, including Hipparchus known for locating things and summarized by Ptolemy
. The second is the literary tradition, starting with Homer, including Hecataeus and summarized by Strabo. A
long period of decline set in during the Middle Ages. When geographic horizons contracted and observations
piled up in Christian monasteries. Then the Age of Exploration began in the late 15th century, and the
geographic horizons were again pushed back . The arrival of all these new innovations and observations in
Europe was enormously stimulating and started a sequence of events that continues to the present. In the first
place, the concepts derived from a literal reading of the scriptures were challenged, and the battle to establish
the principles of what we call academic freedom began . This is the right of professionally qualified scholars
to seek answers to questions, to publish their findings , and to teach what they believe to be the truth. Free
from any controls except the standards of scholarly procedure established within their own professions.
Who belongs to the mathematical traditions of locating places?
1. Home 2. Strabo 3. Hecataeus 4. Hipparchus

97. Who promoted the logic of general to particular procedures?


1.Eratosthenes 2. Homer 3. Aristotle 4. Plato

98. The earliest investigation of geographic questions starts with:


1. Eratosthenes 2. Homer
3. Maps prepared by Sumerians 4. Origin of human scholarship

99. Which one of the followings in not a feature of the Age of Exploration?
1. Promotion of search for truth 2. Formulation of new interpretations
3. Academic restrictions 4. Underming religious beliefs

100. Inductive approach is related with-


1. Deduced knowledge 2. Empirically observed knowledge
3. Philosophical knowledge 4. Theological knowledge

54
PYQ 2023 BSSEI (M-8777279548) Coaching for NET Geography

NTA NET Geography February 2023


1. Which of the following geomorphic phenomena moves fastest?
1. Mud flow 2. Earth flow
3. Debris Avalanche 4. Solifluction

2. How many minimum seismic stations data is required to map the epicentre of earthquake?
1. One 2. Two 3. Three 4. Four

3. Who stated that valley side slope gives a clue to the relative importance of exogenic and endogenic
processes?
1. Davis 2. Penck 3. King 4. Gilbert

4. Which of the following is not directly related to magnitude of earthquake?


1. Length of fault section that broke 2. Damage severity
3. Depth of focus 4. Amount of energy released by earthquake.

5. Which atmospheric gas is primary ingredient of photochemical smog?


1. Carbon dioxide 2. Ozone 3. Argon 4. Methane

6. Which is the correct sequence of height of tropopause (in km) over polar areas, over middle latitudes and
over equator?
1.9, 11, 16 2. 19, 21, 26
3. 5, 8, 20 4. 10, 20, 30

7. From the Earth's heat budget, out of total incoming radiation, what percent is reflected back to space?
1. about 10 percent 2. about 20 percent
3. about 30 percent 4. about 50 percent

8. Which type of lightening has potential to cause more damage to life and property?
1. Inter-cloud lightning 2. Positive cloud-to-ground lightning
3. Negative cloud-to-ground lightning 4. Intra-cloud lightning

9. Which one of the following approaches advocates to think that the most effective philosophical ground
for protecting the natural environment is to think in terms of protecting natural beauty?
1. Deep ecology 2. Enviromental pragmatism
3. Social ecology 4. Enviromental aesthetics

10. Which one of the following has maximum biomass in terrestrial ecosystem?
1. Tertiary consumers 2. Secondary consumers
3. Primary consumers 4. Producers

11. Manufacture of biomass by organisms that eat plants is termed as:


1. Primary productivity 2. Net primary productivity
3. Secondary productivity 4. Tertiary productivity

12. In disaster reduction and management which one of the following is part of post-disaster stage?
1. Vulnerability analysis 2. Disaster prediction
3. Disaster warning 4. Rehabilitation

13. Which one of the following measure represents perhaps the most sensitive barometer of the fitness of the
social environment for human life?
1. Maternal Mortality Ratio 2. Crude death rate
3. Child mortality rate 4. Infant mortality rate

55
PYQ 2023 BSSEI (M-8777279548) Coaching for NET Geography
14. Who among the following first used the term Ecumene and Non-ecumene to distinguish between
inhabited and unhabited portion of earth's surface?
1. Romans 2. Greeks 3. German 4. French

15. Which one of the following geographer famously stated that 'population is the point of reference from
which all the other elements are observed and derive significance and meaning"?
1. J. I. Clarke 2. G. T. Trewartha
3. E. A. Jones 4. W. Zelinsky

16. Which of the following does not represent functions of the town?
1. Culture and recreation 2. Transportation
3. Fishing and farming 4. Resource Extraction

17. Identify the World's leading country in the exports and imports of commercial services in 2021 as per
World Trade Organisation (WTO) report..
1. USA 2. India 3. China 4. Japan

18. Which of the following is not one of the broad five socio-economic themes in Aspirational Districts
Programme?
1. Health and Nutrition
2. Financial Inclusion and Skill Development
3. Basic Infrastructure
4. Import Substitution and Export Promotion

19. Which one of the following highlighted that "An area is poor because it is poor"?
1. Gunnar Myrdal's cumulative causation
2. Perroux's growth pole
3. Meadow's limits to growth
4. Rostow's stages of Economic Growth

20. In which country magnetite ores are mined at Kiruna and Gallivare?
1. Sweden 2. Australia 3. Brazil 4. Canada

21. In 2021, which country was the leading steel producer and exporter of the world ?
1. Japan 2. China 3. USA 4. Australia

22. Technopolies Silicon Forest and Silicon Valley are located near which cities, respectively?
1. Boston and San Francisco
2. Sao Paulo and Seattle
3. Seattle and San Francisco
4. Seattle and Osaka

23. Identify the city from the following where 27th meeting of the Conference of Parties (COP-27) took
place?
1. Glasgow 2. Sharm El-Sheikh 3. Madrid 4. Bonn

24. Considering the food security, arrange the following sub-regions of Africa in descending order according
to the percentage of Prevalence of Undernourishment (POU) (2018).
A. North Africa B. Eastern Africa
C. Western Africa D. Sub-Saharan Africa
1. D-B-A-C 2. B-D-C-A
3. C-D-B-A 4. B-D-A-C

56
PYQ 2023 BSSEI (M-8777279548) Coaching for NET Geography
25. Which of the following books are written by Alfred Thayer Mahan?
A. The Influence of Sea Power and History
B. The Life of Nelson
C. The Influence of Sea Power upon French Revolution and Empire
D The Geographical Pivot of History
1. A and D only 2. C and D only
3. B, C and D only 4. A, B and C only

26. Which of the following country is not considered as a Nation-State?


1. Belgium 2. Japan 3. Egypt 4. Sweden

27. Identify the scholar who laid the foundation of the dichotomy of Systematic vs. Regional Geography.
1. Cook 2. Varenius 3. Kant 4. Darwin

28. Who was the author of 17 Volumes of "Geographical Treaties"?


1. Ptolemy 2. Beatus 3. Strabo 4. Jerome

29. The scientific observations about the size, shape and circumference of the earth were made by
Eratosthenes and Hipparchus at which of the following place?
1. Tanais 2. Tyras 3. Abydos 4. Alexandria

30. Which of the following is not a humanistic geographer?


1. Tuan 2. Kirk 3. Kropotkin 4. Strabo

31. The actual labour productivity of farms was compared with what could have been achieved under
optimizing decision- making by the farmers acting as rational economic persons. Which of the following
geographer conducted this study?
1. Wright 2. White 3. Kirk 4. Wolpert

32. The productivity of rice in Punjab depends on availability of irrigation. Which type of relationship of a
system can be explained by the above given statement?
1. Series relation 2. Parallel relation
3. Feedback relation 4. Complex compound relation

33. Which one of the following sets of book and author is wrongly matched?
1. Human Geography:A welfare Approach - D. M. Smith
2. Geography and Geographers - D. Harvey
3. Radical Geography - R. Peet
4. A Geographical Introduction to History - L. Febvre

34. Which of the following information is used in altimetric frequency graph?


1. Spot height 2. Density of population
3. Contour lines 4. Cumulative frequency

35. Which type of maps are called cartograms?


1. Maps on which very minute details about very small areas like towns and villages are given.
2. The million sheets of the Survey of India.
3. Maps on which diagrams and graphs are superimposed to show the spatial distribution of
data.
4. A rough drawing without scale.

36. Which of the following Indian language group matches correctly with its speech area?
1. North Dravidian – Odisha 2. Dardic Nicobar Islands
3. Mon-Khmer – Assam 4. Munda Jammu and Kashmir

57
PYQ 2023 BSSEI (M-8777279548) Coaching for NET Geography
37. Which one of the following pair is wrongly matched?
Pass/Gap Connected Cities
1. Shencottah gap - Tirunelveli
2. Palghat gap - Coimbatore
3. Burhanpur gap - Itarsi
4. Thal ghat gap - Ahmedabad

38. Which one of the following sets of multipurpose project and associated river is wrongly matched?
1. Matatila -Banas river 2. Banasagar- Sone river
3. Ukai - Tapi river 4. Papanasam - Tamraparni river

39. Which of the following region lies in the lowest seismic sensitive zone in India?
1. Bundelkhand region 2. Gangia region
3. Konkan region 4. Godavari basin

40. Which of the following set is wrongly matched?


1. Kal-Baisakhi - West Bengal
2. Cherry-Blossom - Andhra pradesh
3. Mango showers - Karnataka
4. Bardoli Chheerha - Assam

41. In which of the layers temperature decreases as altitude increases?


A. Thermosphere B. Troposphere
C. Mesosphere D. Stratosphere
Choose the correct answer from the options given below:
1. A and B only 2. B and C only 3. B and D only 4. C and D only

42. Which of the following is/are aquatic ecosystems?


A. Grassland B. Wetland C. Delta D. Forests
Choose the correct answer from the options given below:
1. A only 2. A and B only
3. B and C only 4. Band D only

43. Which of the following are examples of chemical weathering?


A. Movement of rock along fracture B. Rusting of Iron
C. Breaking of Feldspar to clay D. Acid action
Choose the correct answer from the options given below:
1. A and B only 2. B and C only 3. A, B and C only 4. B, C and D only

44. Which of the following statements are true about aerosols in the atmosphere?
A. Aerosols include sea salts, fine soil, smoke, pollens, soot.
B. Aerosols in the air contribute to the various hues of red and orange at sunrise and sunset.
C. The concentration of aerosols is maximum in mesosphere.
D. Aerosols act as surfaces on which water vapour condenses.
Choose the correct answer from the options given below:
1. A and Conly 2. B and C only 3. A, B and D only 4. C and Donly

45. Which of the following statements are NOT true about Spatial Analysis?
A. The application of quantitative techniques in locational analysis is known as 'spatial analysis'.
B. It is not associated with the philosophy of positivism.
C. The generalisation arrived at with the application of spatial analysis is based on: direction, distance and
connection.
D. It got popularity due to deterministic approach in geography.
Choose the correct answer from the options given below:
1. A and Conly 2. A and D only 3. B and D only 4. C and Donly

58
PYQ 2023 BSSEI (M-8777279548) Coaching for NET Geography
46. Which of the following statements are correct?
A. Agenda 21 was adopted in the 1992 Earth Summit held in Rio de Janeiro, Brazil.
B. Montreal protocol was also adopted in the 1992 Earth Summit at Rio de Janeiro, Brazil.
C. The main aim of Montreal protocol was to reduce carbon-di-oxide emissions by member nations.
D. The Commission on Sustainable Development(CSD) was created in 1992 for effective follow-up of
conference held in Rio de Janeiro, Brazil.
Choose the correct answer from the options given below:
1. A and B only 2. A and Conly
3. A and D only 4. C and Donly

47. Which of the following factors may be attributed to the Urban decline?
A. Changes in forms of transport B. Exhaustion of mineral deposits
C. Climate change D. City governance
E. Competition from nearby towns
Choose the correct answer from the options given below:
1. A, B and C only 2. A, B and E only
3. A, B, D and E only 4. A, C and D only

48. Which of the following options correctly represent new additions to improve the data quality by Census
of India, 2011?(Drop)
A. Option of 'others'- other than male/female
B. Tenure status of land under cultivation
C. Four new codes on disability
D. Information about rentiers
Choose the correct answer from the options given below:
1. A, D and E only 2. A, B and D only
3. A, B and E only 4. A, C and D only

49. Which of the following statements are NOT correct?


A. Mahogany, Ebony and Rosewood are main trees of the Tropical Wet Evergreen Forest.
B. Sal and Teak are major species of the Tropical Monsoon Forest.
C. Chestnut tree is known as 'axe-breaker' in Chaco region of Paraguay and southern Brazil.
D. Chinchona tree bark is used to manufacture chewing gum.
E. Tannin is found in the bark of hemlock, mangrove species and teak trees.
Choose the correct answer from the options given below:
1. B, C and D only 2. A, C and Eonly
3. A and B only 4. C, D and E only

50. Which of the following defensive site cities may also be classified as 'hilltop site cities'?
A. Ibadan B. Montreal C. Edinburgh
D. New York E. Hamburg
Choose the correct answer from the options given below:
1. B, D and E only 2. A, B and D only
3. A and C only 4. A, C and E only

51. According to Ullmann the necessary elements of spatial interaction include:


A. Complementarity B. Sustainability
C. Convenience D. Intervening opportunities
E. Transferability
Choose the correct answer from the options given below:
1. A, C and E only 2. C, D and E only
3. A, D and E only 4. A, B and C only

59
PYQ 2023 BSSEI (M-8777279548) Coaching for NET Geography
52. Which of the following pairs are matched correctly?
Cultural region State
A. Hadoti Rajasthan
B. Khandesh Maharastra
C. Baghelkhand Bihar
D. Ruhelkhand Madhya Pradesh
E. Malwa Punjab
Choose the correct answer from the options given below:
1. A, B and C only 2. B, C and D only
3. C, D and E only 4. A, B and E only

53. Which of the following represents main attributes of Jati?


A. Jatis are hierarchically graded.
B. They invariably have a Jati-linked occupation.
C. Members of a Jati generally share heterogenous culture.
D. Consideration of achievements and mobility determine interaction between different units.
E. Jatis are endogamous unit
Choose the correct answer from the options given below:
1. A, B, C and E only 2. A, B and E only 3. A, C, D and E only 4. A, B, D and E only

54. Which of the following criteria is used to study a state by political geographers?
A. Power over the people of an area unrestrained by laws originating from outside the area.
B. Recognition by a significant portion of the international community.
C. Definite portion of earth's land surface with recognized limits.
D. Administrative system to perform functions needed or desired by the people.
E. Organized means of transmitting goods, people, and ideas from one part of territory to another. Choose
the correct answer from the options given below:
1. A, B and C only 2. B, C and D only
3. C, D and E only 4. A, B and E only

55. Which of the following statements are correct about impact of Darwin on Geographical thought?
A. James Hutton demonstrated the theory of uniformitarianism in landform evolution.
B. Fleure used struggle and selection concept in his studies on races, states and towns.
C. Darwin's work on the origin of atolls and the idea regarding slow transformation of biological species
over time, together inspired Davis to develop his own theory of the cycle of erosion.
D. Stoddart stressed the need for physiological study to assess environmental effects on man.
Choose the correct answer from the options given below:
1. A and B only 2. B and C only 3. A and C only 4. C and D only

56. Which of the follwing statements are correct?


A. Davisian model of cycle of erosion envisages space-dependent series of landform development.
B. Penck's model of geographical cycle envisages time independent series of landform development.
C. Davisian geographical cycle over emphasized stage and gave less attention to structure.
D. Penck emphasized that geomorphic forms are expressions of phase and rate of upliftment in relation to
rate of degradation.
Choose the correct answer from the options given below:
1. B, C and D only 2. A, B and C only 3. B and C only 4. C and D only

57. Consider the following statements related to resources and infrastructural development in India.
A. Chromite deposits are found in the form of segregation masses and veins in ultra-basic rocks such as
dunite, peridotite and serpentine.
B. The objectives of Biosphere Reserves in India is to promote research on ecological conservation and
industrial development.
C. Jamnagar oil refinery has the largest installed capacity and uses imported crude petroleum.
D. The ilmenite beach sands of Kerala contains the largest reserves of Monazite in India.

60
PYQ 2023 BSSEI (M-8777279548) Coaching for NET Geography
Choose the correct answer from the options given below:
1. B, C and D only 2. A, B and C only
3. A, B and D only 4. A, C and D only

58. Which of the following correctly represent the main foci of geographical interest in electoral studies?
A. Geography of Symbols B. Geography of Voting
C. Geography of Voting Margins D. Geographic influences on Voting
E. Geography of Representation
Choose the correct answer from the options given below:
1. A, B and E only 2. A, B, D and E only
3. B, C, D and E only 4. B, D and E only

59. Which of the following statements are not correct?


A. Stubble burning contributes to air pollution.
B. Discharge of warm water from thermal power plants into rivers does not contribute to thermal pollution.
C. Noise pollution can cause emotional or psychological effects such as irritability, anxiety and stress.
D. In mountainous areas, terrace farming is the only option to control soil erosion.
Choose the correct answer from the options given below:
1. A and B only 2. B and C only
3. C and D only 4. B and D only

60. Which of the following statements are NOT correct regarding the concept of growth pole?
A. Perroux's original work focuses on the development of growth poles in economic space.
B. Boudeville comprehensively excluded the geographical dimension from growth pole.
C. The processs of polarisation depends upon agglomeration economies.
D. The trickle-down effect are of minimum interest to regional planners.
E. Spatial interactions include movement of goods only.
Choose the correct answer from the options given below:
1. A and C only 2. A, C and D only
3. B, C, D and E only 4. B, D and E only

61. Which of the following countries founded the organisation of the Petroleum Exporting Countries
(OPEC) in 1960?
A. Iran B. Iraq C. Indonesia D. Libya E. Saudi Arabia
Choose the correct answer from the options given below:
1. A, B, D and E only 2. A, C, D and E only
3. A, B and E only 4. A, B, C and E only

62. Which of the following statements are NOT correct?


A. Spatial interaction is reflected in the flow of people, goods and information.
B. Gravity model is used for spatial interaction analysis.
C. Hurst applied concepts of conduction, convection and radiation of physics in his model of spatial
interaction.
D. The direction of spatial interaction is determined exclusively by intervening opportunities.
Choose the correct answer from the options given below:
1. A and B only 2. A, B and C only 3. B, C and D only 4. C and D only

63. Which of the following statements are correct with reference to Weber's Industrial location theory?
A. It is based on the assumption of variable physical and cultural landscape.
B. Entrepreneurs have perfect knowledge of location of raw material and the market.
C. Agglomeration has no role in determining industrial location.
D. There are multiple modes of transportation.
E. It is based on principle of least- cost location
Choose the correct answer from the options given below:
1. B and E only 2. A, B and E only 3. C, D and E only 4. B, C and D only

61
PYQ 2023 BSSEI (M-8777279548) Coaching for NET Geography
64. Consider the following statements related to transport, agricultural and mineral resources in India.
A. Konkan railway connects Roha to Kanyakumari and track measures 760km.
B. Pradhan Mantri Gram Sadak Yojana provides single all weather road connectivity to the habitation of
1000 persons or more.
C. Maharashtra has a longer crushing period of sugarcane than Uttar Pradesh.
D. Andhra Pradesh with the share of more than 98% of total production of mica is the leading mica
producing state in India. Choose the correct answer from the options given below:
1. B and C only 2. A and B only
3. B and D only 4. C and D only

65. Consider the following statements regarding transportation and irrigation in India.
A. Sirhind canal originates from Satluj river to supply water for irrigation in Rajasthan.
B. Kerala state is bestowed with highest length of roads Km per 100 sq.km area among all states of India.
C. Buckingham canal is the part of National Waterway No. 5 of India.
D. Tubewells, as mode of irrigation, provide water to the largest cultivated area of India.
Choose the correct answer from the options given below:
1. B and C only 2. A and D only
3. A and C only 4. B and D only

66. Match List I with List II


List (Local wind) List II (Type)
A. Levanter I. Warm wind in Argentina
B. Norwester II. Cold wind in France
C. Zonda III. Cold wind in Southern Spain
D. Bise IV. Dry and gusty wind in New Zealand
Choose the correct answer from the options given below:
1. AIV, BII, C-I, DIII 2. A - III, B-IV, C-I, D-II
3. A- III, BI, C-II, DIV 4. AII, BIV, C- I, DIII

67. Match List I with List II


List I (Date) List II (Celebrated as)
A. February 2 I. Earth Day
B. April 22 II. World Ozone Day
C. June 5 III. World Wetland Day
D. September 16 IV. World Environment Day
Choose the correct answer from the options given below:
1. A IV, BI, C-II, D-III 2. A-III, B-IV, C-I, D-II
3. A 11, B-1, C-IV, D-III 4. A III, BI, C-IV, D-II

68. Match List I with List II


List I (Consumers/Decomposers) List II (Example)
A. Primary consumers I. Fungi
B. Secondary consumers II. Deer
C. Tertiary consumers III. Fox
D. Decomposers IV. Tiger
Choose the correct answer from the options given below:
1. A-II, B-IV, C-I, D-III 2. A III, B-II, C-IV, D-I
3. A III, BI, C-IV, D-II 4. A-II, B-III, C-IV, D-I

69. Match List I with List II


List I (Work) List II (Scholar)
A. Crop-combination analysis I. Stouffer
B. Isolated state II. Whittlesey, D
C. Intervening opportunities III. Von Thunen
D. Sequent Occupance IV. Weaver, J. C.

62
PYQ 2023 BSSEI (M-8777279548) Coaching for NET Geography
Choose the correct answer from the options given below:
1. A-IV, B-III, C-II, D-I 2. A-1, B-III, C-IV, D-II
3. A-III, B-II, C-I, D-IV 4. A IV, B-III, C-I, D-II

70. Match List I with List II


List I (Concept) List II (Scholar)
A. Primarrumpf I. L.C. King
B. Pediplain II. W. M. Davis
C. Panplane III. W. Penck
D. Peneplain IV. C. H. Crickmay
Choose the correct answer from the options given below:
1. A III, BI, C-II, D-IV 2. A-1, B-IV, C-III, D-II
3. A-II, B-1, C-III, D-IV 4. A III, BI, C-IV, D-II

71. Match List I with List II


List I (Theory about Origin of Tides) List II (Scholar)
A. Dynamic theory I. R. A. Harris
B. Stationary wave theory II. Newton
C. Progressive wave theory III. Laplace
D. Equilibrium theory IV. Willam Whewell
Choose the correct answer from the options given below:
1. A-II, B-III, C-IV, D-I 2. A-III, B-I, C-IV, D-II
3. A III, BI, C-II, D-IV 4. A 1, B III, C-IV, D-II

72. Match List I with List II


List (Tributary) List II (River)
A. Kali Sindh I. Yamuna
B. Hasdeo II. Tapi
C. Sindh III. Chambal
D. Purna IV. Mahanadi
Choose the correct answer from the options given below:
1. A-1, B-IV, C-III, D-II 2. A- III, BII, C- IV, DI
3. A - II, BIV, C- I, DIII 4. A - III, B-IV, C-I, D-II

73. Match List I with List II


List I (Major language) List II (Sub-language)
A. Dardic I. Khasi
B. Mon-Khmer II. Munda
C. North Dravidian III. Shina
D. Austric IV. Gondi

Choose the correct answer from the options given below:


1. A-II, B-I, C-IV, D-III 2. A-III, B-II, C-I, D-IV
3. A IV, B-III, C-II, D-I 4. A III, BI, C-IV, D-II

74. Match List I with List II


List I (Wind) List II (Description)
A. Bores I. North wind, strong and cool with clear skies
B. Eurus II. East wind, warm and gentle
C. Notus III. South wind, wet and sometimes violent
D. Zephyrus IV. West wind, dreaded, balmy with gale force
Choose the correct answer from the options given below:
1. A-II, BI, C-IV, D-III 2. A-1, B-II, C-III, D-IV
3. A-IV, B-III, C-II, D-I 4. A III, BIV, C-I, D-II

63
PYQ 2023 BSSEI (M-8777279548) Coaching for NET Geography
75. Match List I with List II
List I (Statement)
A. Regional geographers may perhaps be trying to put boundaries that do not exist around areas that do not
matter.
B. Innovation of diffusion analysis represent perhaps the most successful incorporation of time into geography
outside of historical geography.
C. "To identify geographical problems,........ perhaps to solve them."
D. "Historical processes......... may provide the basis for structuring of landscape as a way of life."
List II (Expert)
I. Norton II. Ley III. Kimble IV. Stoddart
Choose the correct answer from the options given below:
1. A III, BI, C-IV, D-II 2. A 1, B II, C-IV, D-III
3. A-II, B-III, C-I, D-IV 4. A IV, BII, C-I, D-III

76. Arrange the following principles/concepts/theories of geomorphology in chronological order (old to


recent) in terms of inception.
A. Principle of Uniformitarianism B. Dynamic Equilibrium Theory
C. Die Morphologische Analyse D. Geographical cycle of erosion
Choose the correct answer from the options given below:
1. A, B, C, D 2. C, A, D, B 3. A, D, C, B 4. A, C, D, B

77. Identify the correct sequence of the food chain in a marsh community.
A. Butterfly B. Hawks C. Snake D. Green plants E. Frogs
Choose the correct answer from the options given below:
1. C, B, D, E, A 2. D, A, B, E, C
3. D, A, E, C, B 4. D, E, A, C, B

78. Identify the correct sequence of the concentric zones of different land uses from the city center in Von
Thunen's model?
A. Crop farming without fallow B. Firewood and timber
C. Market gardening and milk production D. Three-field system
E. Livestock ranching
Choose the correct answer from the options given below:
1. B, C, D, A, E 2. C, D, B, A, E 3. C, B, A, D, E 4. C, D, E, B, A

79. Arrange the following countries in a descending order of their Human Development Index (HDI), 2021
ranking.
A. Australia B. Norway C. Switzerland D. Iceland
Choose the correct answer from the options given below:
1. B, C, D, A 2. B, C, A, D 3. C, B, A, D 4. C, B, D, A

80. Arrange the following regions (in descending order) in terms of proportion of non-religious population in
the world.
A. Anglo-America B. South-Pacific
C. East Asia D. Eastern Europe E. Latin America
Choose the correct answer from the options given below:
1. D, C, B, E, A 2. C, D, B, A, E
3. D, C, E, B, A 4. C, D, A, E, B

81. Arrange the following publications in chronological order


A. The Nature of Geography B. Critique of Pure Reason
C. Origin of Species D. Exceptionalism in Geography
Choose the correct answer from the options given below:
1. B, C, A, D 2. A, C, B, D
3. C, D, B, A 4. D, A, C, B

64
PYQ 2023 BSSEI (M-8777279548) Coaching for NET Geography
82. Which is the correct sequence of radiation from longest to shortest wavelengths?
A. X-rays B. Visible C. Infrared D. Ultraviolet
Choose the correct answer from the options given below:
1. C, D, B, A 2. C, B, D, A
3. D, A, B, C 4. C, A, D, B

83. Identify the sequence of clouds in sky (first to last) as a warm front approaches an observer/ place in the
middle latitudes.
A. Cirrostratus B. Nimbostratus C. Altostratus D. Cirrus
Choose the correct answer from the options given below:
1. D, A, C, B 2. C, D, B, A
3. D, C, A, B 4. D, B, C, A

84. Identify the correct sequence of the major salts (in percentage) in the ocean waters in descending order.
A. Magnesium chloride B. Calcium carbonate
C. Sodium chloride D. Magnesium sulphate
Choose the correct answer from the options given below:
1. B, D, C, A 2. C, A, D, B
3. C, D, B, A 4. D, C, A, B

85. Arrange the longitudinal fault lines of Himalyas from south to north.
A. Himalayan Frontal Fault (HFF) B. Main Boundary Thrust (MBT)
C. Indus-Tsang Po Suture Zone (ITSZ) D. Main Central Thrust (MCT)
Choose the correct answer from the options given below:
1. A, B, C, D 2. A, B, D, C 3. B, C, A, D 4. D, B, A, C

86. Given below are two statements:


Statement I: Maximum salinity is seen at Tropic of Cancer and the reason may be low rainfall, high
evaporation and high atmospheric pressure.
Statement II: Minimum salinity is seen in temperate zone because of excess of evaporation and larger supplies
of fresh water.
In light of the above statements, choose the correct answer from the options given below:
1. Both Statement I and Statement II are true.
2. Both Statement I and Statement II are false.
3. Statement I is true but Statement II is false.
4. Statement I is false but Statement II is true.

87. Given below are two statements, one is labelled as Assertion (A) and the other is labelled as Reason (R).
Assertion (A): El Nino has a strong effect on marine life along the Peru coast.
Reason (R): During El Nino, upwelling of nutrient rich cold water is weakened which affects phytoplankton.
In light of the above statements, choose the correct answer from the options given below:
1. Both (A) and (R) are true and (R) is the correct explanation of (A).
2. Both (A) and (R) are true but (R) is NOT the correct explanation of (A).
3. (A) is true but (R) is false.
4. (A) is false but (R) is true.

88. Given below are two statements about Structuralism/ Structuralists:


Statement I: Structuralism is concerned with discovering the things that are real and important in social life
by looking beneath the taken-for-granted socio-economic categories.
Statement II: Structuralist appreciated the human creativity and intentionality related to individual's lives.
In light of the above statements, choose the most appropriate answer from the options given below:
1. Both Statement I and Statement II are correct.
2. Both Statement I and Statement II are incorrect.
3. Statement I is correct but Statement II is incorrect.
4. Statement I is incorrect but Statement II is correct.

65
PYQ 2023 BSSEI (M-8777279548) Coaching for NET Geography
89. Given below are two statements, one is labelled as Assertion (A) and the other is labelled as Reason (R).
Assertion (A): Landscapes dominated by a collection of folk customs change relatively little over time,
whereas modern communications facilitate frequent changes in popular customs.
Reason (R): Folk culture is more likely to vary from time to time at a given place where as popular culture is
more likely to vary from place to place at a given time.
In light of the above statements, choose the most appropriate answer from the options given below:
1. Both (A) and (R) are correct and (R) is the correct explanation of (A).
2. Both (A) and (R) are correct and (R) is NOT the correct explanation of (A).
3. (A) is correct but (R) is not correct.
4. (A) is not correct but (R) is correct.

90. Given below are two statements, one is labelled as Assertion (A) and the other is labelled as Reason (R).
Assertion (A): Insular countries are geopolitically powerful states.
Reason (R): Geographical location, advantage of navigational facilities and access to resources are the unique
advantages of insular countries.
In light of the above statements, choose the most appropriate answer from the options given below:
1. Both (A) and (R) are correct and (R) is the correct explanation of (A).
2. Both (A) and (R) are correct and (R) is NOT the correct explanation of (A).
3. (A) is correct but (R) is not correct.
4. (A) is not correct but (R) is correct.

Read the following passage and answer questions: (91-95)


Risk is a product of two components: the hazard, which can be defined as the probability of occurrence of a
specific event within a given time period; and the consequence, typically measured by the cost of damage or
loss of life. Several environmental and human factors contribute to landslide hazard, principally by
determining the susceptibility of a slope or geological system to instability. These preparatory factors include
slope geometry, hydrology (surface and groundwater), soil type, underlying geology, vegetation and slope
loading. Landslide triggering factors include rainfall, earthquakes, volcanic activity, changes in groundwater,
erosion or undercutting and human activities (e.g. mining, construction or deforestation). Hydrological
processes can exert a significant and dynamic influence on slope stability, as both a landslide preparatory
factor, in terms of pore pressure and groundwater levels within the slope, and as a landslide trigger factor due
to rainfall infiltration or pore pressure changes. Depending on the hydrological properties of the slope material,
an increase in moisture content and pore-water pressure can reduce the mechanical strength of the slope to the
point of failure. Landslide consequences depend on the type and magnitude of the landslide; the process of
emplacement or runout, which determines the rate, travel distance and volume of debris (the hazard footprint);
and the exposure and vulnerability of elements within the area of impact. Landslides can destroy or bury
buildings, infrastructure and crops, cut off transport links, water and power supplies, and result in significant
direct and indirect economic losses.
91. What do you mean by hazard foot-print of landslides?
1. Process of emplacement.
2. Increase in moisture content.
3. Travel distance and volume of debris triggered by a landslide.
4. Hydrological property of area.

92. The consequences of landslide mainly depend upon:


1. Type and mangitude of landslide.
2. Low intensity and low frequency of landslide.
3. Space and time of occurence of landslide.
4. Instability and slope loading of the area.

93. Which process exerts a significant and dynamic influence on slope stability?
1. Underlying geology. 2. Slope geometry.
3. Slope loading. 4. Hydrological process.

66
PYQ 2023 BSSEI (M-8777279548) Coaching for NET Geography
4. Which are the triggering factors for occurence of landslides?
1. Slope geometry, soil type and slope loading.
2. Rainfall, earthquake and changes in ground water.
3. Hydrology, underlying geology and vegetation.
4. Slope geometry, rainfall and soil type.

95. Geomorphic risk is defined as:


1. Hazard in a given space and exposure.
2. Hazard in a given time and adaptation.
3. Probability of occurence of extreme event in the given time and loss of life and property.
4. Vulnerability in given time and occurence of hazard.

Read the following passage carefully and answer questions: (96-100)


Thomas Malthus was an English clergyman who, in 1798, published an Essay on the Principles of Population
in which he put forward the view that, "the power of population is indefinitely greater than the power of the
earth to produce subsistence for man." He thought that a balance could only be maintained if famine, disease
or war periodically increased the death rate and reduced population growth. His pessimistic ideas were
accepted by several other 19th century scholars in England and France. At the end of the 19th century, the
population of England was only about 10 million, but much of their food supply had to be produced from the
limited agricultural land of the country. Changes in land tenure, brought about by enclosure of the old common
fields and the formation of large farms in the place of small scattered plots led to rural depopulation. The
towns, especially those where the new factory industries had been established, grew very rapidly and were
overcrowded, dirty and unhealthy. The people who lived in them were poor, under-fed, overworked and had
little resistance to disease. Thus, had food supply been reduced or population expanded too rapidly, these
people would have suffered starvation and epidemics would have reduced the population. This has already
happened twice during England's history; the Black Death of fourteenth century and the Great Plague of the
seventeenth century coincided with periods when harvest was bad and there were food shortages. Hunger
reduced resistance to diseases and bubonic plague caused the death of many thousands of people. Malthus
was afraid that something similar would happen again. In his time, great advances were being made in the
treatment and control of diseases such as cholera, typhoid and smallpox which were still rife in England and
Europe. This meant that death rates, and particularly infant mortality rates, were falling. Malthus calculated
that population could double every twenty-five years, but no similar increases in food supplies could be
expected. He could not have foreseen the tremendous changes which were to take place in the nineteenth and
twentieth century.
96. According to Malthus, what is the relationship between population growth and earth's resources?
1. Population grows rapidly as compared to subsistence produced from earth.
2. Population growth is not related with earth's resources.
3. Population growth may be checked by famine, disease or war.
4. Population doubles every 25 years.

97. Which one of the following options is considered as the best description of Malthusian perspective related
to population growth and resources?
1. Optimistic 2. Neutral 3. Pessimistic 4. Critical

98. The Great Plague in England's history occured in which period?


1. Fourteenth century 2. Thirteenth century 3. Seventeenth century 4. Fifteenth century

99. What factor(s) was/were responsible for the deaths of people during Black Death and Great Plague in
England?
1. Starvation 2. Plague 3. Poor medical infrastructure 4. Starvation and Plague

100. Improvement in the treatment and control of diseases in England and Europe helped to substantially
reduce mortality in which age group of the population?
1. Adult population 2. Infant population
3. Older population 4. Older children

67
PYQ 2023 BSSEI (M-8777279548) Coaching for NET Geography

NTA NET Geography June 2023


1. Delta formed by the Ganga river is an example of______.
1. Bird foot delta 3. Cuspate delta
2. Estuarine delta 4. Arcuate delta

2. In which of the following countries of the world, the farming is severely restricted by climate?
1. Mexico 3. Canada
2. Spain 4. China

3. The approach of functional organization of space was propounded by which one of the following schools
of thought?
1. British School 3. German School
2. French School 4. American School

4. Which one of the following colours is not a part of the rainbow?


1. Violet 3. Orange
2. Grey 4. Red

5. The functioning of ecosystem depends on____


1. Energy flow 3. Biological cycles
2. Nutrient cycles 4. Tidal energy

Choose the most appropriate answer from the options given below;
1. A and D only 3. A,B and D only
2. B and D only 4. A,B and C only

6. The long-term objective of the National Population Policy (2000) of India was to achieve population
stabilization by the year___
1. 2045 3. 2075
2. 2030 4. 2047

7. The standard error of mean is calculated on the basis of___


A. Grand Mean D. Sampling distribution of mean
B. Size of the sample E. Population mean
C. Sample standard deviation
Choose the correct answer from the options given below:
1. D and E only 3. A and D only
2. B and C only 4. B and E only

8. Arrange the following elements of hill slope in correct sequence from top to bottom.
A. Waxing slope C. Debris slope
B. Wanning slope D. Free face
Choose the correct answer from the options given below.
1. A,C,D,B 3. B,A,C,D
2. A,D,C,B 4. C,D,A,B

9. Which of the following statements are correct?


A. Griffith Taylor gave a classification of cities based on age
B. Harris used pure statistical method of standard deviation for functional classification of cities.
C. Sector model of Hoyt was based on the study of spatial patterns of house rents in American cities
D. Chandigarh was planned by Le Corbusier and Canberra by Walter Burley Griffin.
Choose the most appropriate answer from the options given below
1. A,B and C only 3. B,C and D only
2. A,C and D only 4. B and D only

68
PYQ 2023 BSSEI (M-8777279548) Coaching for NET Geography
10. The status of salinity in Indian Ocean (in northern summer) is given below. Identify the correct
statements.
A. The salinity in the Indian Ocean increases from equator to the Bay of Bengal
B. The salinity in the Indian Ocean decreases from equator towards the Bay of Bengal
C. The salinity in the Indian Ocean remains same in different parts
D. The 45% isohaline the divide the Indian Ocean into eastern and western section.
E. Salinity is more in the Arabian Sea than the Bay of Bengal
Choose the correct answer from the options given below:
1. B and E only 3. C and E only
2. A and B only 4. D and E only

11. Who among the following scholars strongly advocated the idea of possibilism?
A. Ratzel and Ritter D. Sauer and Huntington
B. Brunches and Sauer E. Semple and Ratzel
C. Febvre and Blache
Choose the correct answer from the options given below:
1. A and B only 3. C and D only
2. B and C only 4. C and E only

12. Which of the following statements are correct regarding welfare geography?
A. It was revolutionary in theory and revolutionary in practice
B. The basis of welfare approach is on “who gets what, Where and how”
C. It was based on positivism
D. It is a pragmatic approach to overcome the problems of inequalities
Choose the most appropriate answer from the options given below
1. A and B only 3. B and D only
2. B and C only 4. C and d only

13. Landforms formed by erosional activities


A. Yardang and Stumps D. Blow Hole and Seif dune
B. Cirque and Barchan E. Structural bench and Col
C. Arete and Esker
Choose the correct answer from the options given below
1. A and E only 3. C and D only
2. B and C only 4. A and C only

14. Match list I with list II


LIST I LIST II
(Weather parameter) (Instrument)
A. Duration of sun shine I. Hygrograph
B. Atmospheric pressure II. Pan Evaporimeter
C. Relative Humidity III. Campbell-stockes recorder
D. Evaporation IV. Barometer
Choose the correct answer from the options given below:
1. A-II, B-IV, C-III, D-I 3. A-III, B-IV, C-I, D-II
2. A-III, B-IV, C-II, D-I 4. A-IV, B-III, C-II, D-I

15. Which of the following statements are correct?


A. The gap between rural and urban quality of life is not much wide in India
B. About 70% of the population lives in villages in India
C. Indian rural society is still semi-feudalistic.
D. Proportion of rural population is highest among states in Bihar
Choose the correct answer from the options given below
1. A and B only 3. C and D only
2. B and C only 4. A and D only

69
PYQ 2023 BSSEI (M-8777279548) Coaching for NET Geography
16. Bay of Fundy is located along the coast of___
1. Australia 3. Chile
2. Norway 4. Canada

17. The migrants of second wave of Indian Diaspora went to settle in which of the following country?
1. Mauritius 3. Thiland
2. Fiji 4. Canada

18. Who wrote the book ‘Illustration of the Huttonian Theory of the Earth?
1. James Hutton 3. Vidal de la Blache
2. Bowman 4. John Playfair

19. Merit List I with List II


LIST I LIST II
(Author) (Book Title)
A. P.Knox and S.Pinch I. The cultural landscape: An Introduction to Human
Geography
B. Emrys Jones and John Eyles II. Social Geography
C. A.Ahmad III. Urban Social Geography: An Introduction
D. J.M.Rubenstein IV. An Introduction to social Geography
Choose the correct answer from the options given below
1. A-I, B-II, C-III, D-IV 3. A-III, B-IV, C-II, D-I
2. A-II, B-III, C-IV, D-I 4. A-IV, B-III, C-I, D-II

20. Merit List I with List II

LIST I LIST II
(Type of Industry) (Place)
A. Jute I. Kullu
B. Wool II. Mysore
C. Silk III. Hisar
D. Cotton IV. Ongole
Choose the correct answer from the options given below:
1. A-II, B-IV, C-I, D-III 3. A-IV, B-III, C-II, D-I
2. A-IV, B-I, C-II, D-III 4. A-IV, B-I, C-III, D-II

21. Match List I with List II


LIST I LIST II
(Elements of Visual (Feature Identification)
Images)

A. Tone I. To distinguish zones of land use


B. Size II. Land cover and land use
C. Shape III. Orchards
D. Pattern IV. Meander belts
Choose the correct answer from the options given below
1. A-IV, B-I, C-II, D-III 3. A-III, B-II, C-I, D-IV
2. A-I, B-IV, C-III, D-II 4. A-II, B-I, C-IV, D-III

22. The density of oceanic water is generally lowest in____


1. Equatorial region 3. Temperate region
2. Sub-tropical region 4. Polar region

70
PYQ 2023 BSSEI (M-8777279548) Coaching for NET Geography
23. Which of the following areas are associated with South Central Highland?
A. Bundelkhand upland C. Rohikhand plains
B. Vindhya Scarplands D. Narmada Valley
Choose the correct answer from the options given below:
1. A and B only 3. B and C only
2. B and D only 4. C and D only

24. Given below are two statements:


Statement I: The sea waves of seismic origin in general, behave like long gravitational wave.
Statement II: The sea waves of shorter length will reach their maximum height fast, whereas a longer wave
will take a long time.
In the light of the above statements, choose the most appropriate answer from the options given below:
1. Both Statement I and II are correct
2. Both statement I and II are incorrect
3. Statement I is correct but II is incorrect
4. Statement I is incorrect but II is correct

25. Which one of the following is responsible for the blue colour of the sky?
1. Rayleigh Scattering 3. Nonselective scattering
2. Mie scattering 4. Ground radiance

26. Pediplains and Inselbergs are features of the old stage of which one of the following cycles of erosion?
1. Marine Cycle 3. Glacial cycle
2. Eolian cycle 4. Karst cycle

27. Which of the following are not probability sampling techniques?


1. Simple Random 4. Cluster
2. Stratified 5. Convenience
3. Snowball
Choose the correct answer from the options given below:
1. A and E only 3. B and D only
2. C and E only 4. B and C only

28. Which of the following statements with respect to altitude of the sun and spacing of tall buildings is most
relevant?
A. Buildings are usually built far apart in high latitudes.
B. Buildings are usually built closer together in tropics.
C. Buildings are usually built closer together in high latitudes.
D. Buildings are usually built far apart in tropics.
Choose the correct answer from the options given below.
1. A and D only 3. C and D only
2. B and C only 4. A and B only

29. Arrange the following seas in ascending order of concentration of salinity ( minimum to maximum )
A. Red sea C. Baltic sea
B. China Sea D. Black sea
Choose the correct answer from the options given below.
1. A,B,C,D 2. D,C,B,A 3. B,C,A,D 4. C,D,B,A

30. Producers in the trophic level include:


A. Photosynthetic green plants C. Chemosynthetic bacteria
B. Photosynthetic bacteria D. Frogs
Choose the most appropriate answer from the options given below.
1. A,B and D only 3. A,B and C only
2. B and D only 4. B,C and D only

71
PYQ 2023 BSSEI (M-8777279548) Coaching for NET Geography
31. Match List I with List II

LIST I LIST II
(Sectors of Economy) (Name of activity)
A. Primary I. Singing
B. Secondary II. Postal services
C. Tertiary III. Brick-making
D. Quarternary IV. Pearl culture

Choose the correct answer from the options given below.


1. A-I, B-II, C-III, D-IV 3. A-III, B-IV, C-I, D-II
2. A-II, B-III, C-IV, D-I 4. A-IV, B-III, C-II, D-I

32. Read the given statements about Tide producing forces carefully and identify the correct statements.
A. The attraction of the moon is maximum at the point which is closest to the moon on the earth’s
surface.
B. Distance is very important variable.
C. The attraction of the moon is minimum at the point which is closest to the moon on the earth’s
surface.
D. The tide-generating forces are very large in magnitude compared to the gravitational forces.
Choose the correct answer from the options given below.
1. A and B only 3. C and D only
2. B and C only 4. A and D only

33. Given below are two statements:


Statement I: Highest concentration of scheduled caste population is in the Satluj-Ganga plains of India.
Statement II: In terms of absolute number Christians are most numerous in Kerala state of India.
Choose the correct answer from the options given below.
1. Both Statement I and II are true 3. Statement I is true but II is false
2. Both Statement I and II are false 4. Statement I is false but Statement II is true

34. Match list I with list II


LIST I LIST II
(States) (Change in effective Literary Rate, 2001-2011)
A. Kerala I. 16.8
B. Mizoram II. 14.6
C. Himachal Pradesh III. 3.8
D. Bihar IV. 3.0
Choose the correct answer from the options given below.
1. A-I, B-II, C-IV, D-III 3. A-IV, B-III, C-II, D-I
2. A-II, B-I, C-III, D-IV 4. A-III, B-II, C-I, D-IV

35. Which one of the following was introduced by W.Arnold?


1. Spatial processes of plant ecology in settlement dispersion study
2. Monte Carlo technique to the evolution of settlement
3. Utopian social and political thinking affect on settlement network
4. Place name analysis in the settlement study

36. Who among the followings enunciated four stages in the evolution of space economy?
1. John Friedmann 3. Ronald F Abler
2. Karl Marx 4. August Losch

72
PYQ 2023 BSSEI (M-8777279548) Coaching for NET Geography
37. Arrange the following social-ecological movements in correct chronological order.
A. Aarey Movement, Mumbai C. Narmada bachao Andholan
B. Chipko Movement D. Jungle Bachao Andholan
Choose the correct answer from the options given below.
1. A,B,C,D 3. B,C,D,A
2. B,D,C,A 4. D,C,B,A

38. Which one of the following surfaces has the maximum albedo?
1. Dense cloud 3. Bare rock
2. Desert 4. Tropical forest

39. Which one of the following models is based on the study of human ecology?
1. Concentric zone model by Burgess
2. Innovation Diffusion model by Hagestrand
3. Mobility Transition model by Zelinsky
4. Location model by Weber

40. The characteristics of ocean water include:


A. The distribution of ocean currents is not affected by the distribution of density of the oceanic water.
B. Heat lost by the tropical oceans is the major source of heat needed to drive the atmospheric
circulation.
C. Most of the heat absorbed by the oceans in tropics is released as water vapour in polar areas.
D. Less salty waters are at high latitudes.
Choose the correct answer from the options given below.
1. A and B only 3. C and D only
2. B and D only 4. A and C only

41. Given below are two statements: One is labelled as Assertion A and the other is labelled as Reason R:
Assertion (A): With global warming, organisms that were formally restricted to warmer regions will
become more common towards the poles.
Reason (R): Geographic distribution of organisms could not be significantly altered by climate change.
Choose the most appropriate answer from the options given below.
1. Both A and R are correct and R is the correct explanation of A
2. Both A and R are correct but R is not the correct explanation of A
3. A is correct but R is not correct
4. A is not correct but R is correct

42. Organisms with ‘R-selected’ growth patterns tend to occupy which tropic level in their ecosystems?
1. Low trophic level 3. Middle trophic level
2. High trophic level 4. Very high trophic level

43. Cultural Eutrophication is caused by which of the following?


1. Sediments brought by a tributory stream
2. Nutrients brought by a tributory stream
3. Nutrients added into the water bodies by anthropogenic factors
4. Due to abundance of Plankton organisms

44. Arrange the following in chronological order


A. Conference of Parties (CoP-21)
B. Earth Summit
C. World summit on Sustainable Development
D. Montreal Protocol
Choose the correct answer from the options given below.
1. B,D,C,A 3. D,B,C,A
2. D,B,A,C 4. B,C,D,A

73
PYQ 2023 BSSEI (M-8777279548) Coaching for NET Geography
45. Which statements are true about the benefits of World Trade Organization (WTO)?
A. Trade disputes between and among countries are handled constructively
B. Liberalized global trade provides greater choice of products and their qualities
C. Rules make life difficult to all
D. The liberalized trade cuts the cost of living
E. This system promotes conflict
Choose the correct answer from the options given below.
1. A,B and D only 3. C,D and E only
2. A,B and E only 4. B,C and D only

46. When did the last Ice age on Earth take place?
1. Pliocene period 3. Pleistocene period
2. Miocene period 4. Silurian period

47. Buran stroam blows in which part of the world?


1. North America 3. South Africa
2. South America 4. Russia

48. Given below are two statements: One is labelled as Assertion A and the other is labelled as Reason R:
Assertion (A): Christaller developed network of hexagonal complementary areas in central place theory.
Reason (R): The circular complementary areas were having gaps and overlaps.
Choose the correct answer from the options given below.
1. Both A and R are true and R is the correct explanation of A
2. Both A and R are true but R is not the correct explanation of A
3. A is true but R is false
4. A is false but R is true

49. Arrange the following according to increasing wave length.


A. Infrared B. Ultraviolet C. Microwave D. Gamma
Choose the correct answer from the options given below.
1. B,D,C,A 2. D,C,B,A 3. C,A,D,B 4. D,B,A,C

50. The most widely accepted scheme of cultural realms of the world was devised by____
1. Broek and Webb 3. Haggett and Chroley
2. Spencer and Malthus 4. Jones and Gregory

51. Landforms associated with depositional activity:


A. Sea caves and Caldera D. Arches and Plunge-pool
B. Eskers and wave built platform E. Barrier beach and loess
C. Playa and Meander
Choose the correct answer from the options given below.
1. A and B only 2. A and C only 3. B and E only 4. C and E only

52. Which one of the following nomadic tribes lives in northern Finland?
1. Yukaghir 2. Lapps 3. Warlpiri 4. Kirghiz

53. Which of the following statements are NOT true about inter-state distribution of scheduled caste
population in India (2011)?
A. Bihar has the largest number of scheduled caste population among all the major states of India
B. Punjab has the highest percentage share of scheduled casts in its total population.
C. Maharashtra has the third largest population size of scheduled castes in India
D. Tamil Nadu has more scheduled caste population than Andhra Pradesh
Choose the most appropriate answer from the options given below.
1. A and B only 3. C and D only
2. B and C only 4. A and C only

74
PYQ 2023 BSSEI (M-8777279548) Coaching for NET Geography
54. Which one of the following is located near Greenland?
1. Telegraph Plateau 3. Puerto Rico deep
2. Walvis ridge 4. Challenger rise

55. Which of the following is used to represent the geographical data on chorochromatic maps?
1. Colours 3. Points
2. Lines 4. Pictures

56. Arrange the following states of India in ascending order with respect to density of population as per
2011 Cencus.
A. Rajasthan C. Kerala
B. Odisha D. Uttar Pradesh
Choose the correct answer from the options given below.
1. A,B,D,C 3. C,D,B,A
2. B,A,C,D 4. B,C,D,A

57. Which of the following are true for India’s National Health Policy, 2017?
A. Increase of life expectancy at birth to 70 years by 2025
B. Increase of health expenditure by Government to 5.0 % of GDP by 2025
C. Reduction in infant mortality rate to 28 by 2019
D. Reduction of Maternal Mortality rate to 200 by 2020
E. Reduction of total fertility rate (TFR) to 2.1 national and sub-national level by 2025
Choose the correct answer from the options given below.
1. A,B and C only 3. C,D and E only
2. B,C and D only 4. A,C and E only

58. Which of the following statements are true about crop-combination regions?
A. Provides a good basis for agricultural regionalization
B. Helps in studying agricultural productivity
C. Helps in formulation of agricultural policy
D. Provides a comprehensive understanding of agricultural mosaic of an agro-climatic region for
planning and development of agriculture
Choose the most appropriate answer from the options given below.
1. A and B only 3. B,C and D only
2. A,C and D only 4. A,B and C only

59. Which among the following is the correct sequence of the stages through which a new boundary under
most conditions is determined?
A. Allocation B. Delimitation C. Maintenance D. Demarcation
Choose the correct answer from the options given below.
1. D,A,B,C 2. C,B,D,A 3. A,B,D,C 4. A,C,B,D

60. Which of the following facts are associated with photoperiod?


A. Longer the photoperiod the greater is the total possible insolation
B. At poles day and night are always equal
C. At poles the daily photoperiod reaches a maximum of 24 hours in summer and a minimum of zero
hours in winter
D. At poles the daily photoperiod reaches a maximum of 24 hours in winter and a minimum of zero
hour in summer.
Choose the most appropriate answer from the options given below.
1. A and C only 3. B and C only
2. A and B only 4. C and D only

75
PYQ 2023 BSSEI (M-8777279548) Coaching for NET Geography
61. Wirth is known for which concept?
1. Primate City 3. Central Place
2. Rural Urban Continuum 4. Conurbation

62. Which of the following statements are NOT true about regional distribution of Muslim population in
India (2011)?
A. Lakshadweep has the highest proportion of Muslim population
B. Proportion of Muslim population in total population of Uttar Pradesh is higher than in West Bengal.
C. Punjab has the lowest proportion of Muslim population in comparison to other states of India.
Choose the correct answer from the options given below.
1. A,B and C only 3. B,D and E only
2. B,C and D only 4. C,D and E only

63. Kumjawng pass is located in____


1. Sikkim 3. Tripura
2. Arunachal Pradesh 4. Meghalaya

64. The idea/concept of ‘spatial margins of profitability’ in context of industrial location theories was
propounded by____
1. A Losch 3. D.M. Smith
2. Pred 4. Walter Isard

65. Which of the following map making technique uses the intervals in arithmetic progression?
1. Chorochromatic 3. Isarithmic
2. Dot 4. Proportional circle

66. Match List I with List II


List I List II
(Terminology) (Related Aspects)
A. Biogeochemical cycles I. Ecosystem productivity
B. Flow of energy II. Nutrient cycles
C. Biomass III. Trophic Pyramid
D. Food chain IV. Law of Thermodynamics
Choose the correct answer from the options given below.
1. A-IV,B-II,C-I,D-III 3. A-II,B-IV,C-I,D-III
2. A-III,B-I,C-II,D-IV 4. A-I,B-III,C-IV,D-II

67. Aurora Australis pertaining to ionosphere are related to which hemisphere/region?


1. Northern Hemisphere 3. Southern Hemisphere
2. Greenland 4. Canada

68. The Mean (X) of given normally distributed scores is 40. Arrange the following score intervals in
ascending order on the basis of frequency of their classes.
A. 30-50 B. X to 55 C. 30 to X D. 10 to X
Choose the correct answer from the options given below.
1. A,B,C,D 2. B,C,A,D 3. D,B,C,A 4. C,B,A,D

69. In which of the following principles of regional planning each region is considered as a sub-system of
the regional system whole?
1. The principle of vertical unity of phenomena
2. The principle of horizontal spatial unity
3. The principle of space-time continuum
4. The principle of comprehensive development

76
PYQ 2023 BSSEI (M-8777279548) Coaching for NET Geography
70. Which one of the following represents average temperature of water at bottom in the Red sea?
1. 10 degree C 2. 21 degree C 3. 30 degree C 4. 40 degree C

71. Who wrote the essay entitled ‘Trying to Solve the Monsoon Riddle’ in relation to mechanism of
monsoon?
1. Rama Sastry 2. Menon 3. Krishna 4. Parthasartahy

72. As per the Census of India 2011, arrange the following states in ascending order of sex ratio (minimum
to maximum)
A. Uttar Pradesh C. Karnataka
B. Maharashtra D. Himachal Pradesh
Choose the correct answer from the options given below.
1. A,B,C,D 3. C,D,B,A
2. B,C,D,A 4. D,A,C,B

73. Match List I with List II


LIST I LIST II
(Composition) (Plains)
A. A plain largely composed of recent alluvium I. Outwash Plain
B. Deep Sea plain II. Loess Plain
C. A plain formed by wind action III. Flood Plain
D. A plain formed due to deposition of glacial sediments IV. Abyasal plain
Choose the correct answer from the options given below.
1. A-III,B-IV,C-II,D-I 3. A-I,B-IV,C-II,D-III
2. A-II,B-III,C-IV,D-I 4. A-IV,B-II,C-III,D-I

74. Which of the following statements are relevant with respect to Person’s Product-Moment Correlation
coefficient?
A. Both variables have come from the normally distributed population
B. Rxy is a non-parametric measure of the relationship
C. It is known as the covariance of the two variables
D. It is highly suitable to measure relationship of attributes like ‘preferences’ ‘efficiency’ and
‘intelligence’
Choose the most appropriate answer from the options given below.
1. A and B only 2. B and C only 3. A and C only 4. C and D only

75. Match List I with List II


LIST I LIST II
(Warm Ocean Current) (Meeting cold ocean current)
A. Brazil current I. Oyashio Current
B. Gulf stream II. Falkland current
C. Kuroshio current III. Labrador current
D. Agulhas current IV. West wind drift
Choose the correct answer from the options given below
1. A-I,B-II,C-III,D-IV 3. A-IV,B-III,C-II,D-I
2. A-III,B-IV,C-I,D-II 4. A-II,B-III,C-I,D-IV

76. As per the Census 2011, which of the following recorded second and first rank in population growth
during 2001-2011?
A. Arunachal Pradesh D. Jammu and Kashmir
B. Meghalaya E. Bihar
C. Mizoram
Choose the correct answer from the options given below:
1. A and B only 2. B and C only 3. C and D only 4. D and E only

77
PYQ 2023 BSSEI (M-8777279548) Coaching for NET Geography
77. Who among the following were the leading advocates of quantitative revolution in Geography?
A. Semple and Ratzel D. Kansky and Ratzel
B. Mackinder and Ritter E. Hagerstrand and Chorley
C. Peter Hagget and William Bunge
Choose the correct answer from the options given below.
1. A and B only 3. C and D only
2. B and E only 4. C and E only

78. Nokrek Biosphere Reserve is located in____


1. Odisha 3. West Bengal
2. Assam 4. Meghalaya

79. Given below are two statements:


Statement I: Icebergs are not common along the Norwegian coast.
Statement II: Icebergs are very common along the Siberian coast
In the light of the above statements, choose the correct answer from the options given below:
1. Both Statement I and II are true 3. Statement I is true but II is false
2. Both statement I and II are false 4. Statement I is false but II is true

80. Who described State as an organism?


1. Humboldt 3. Mackinder
2. Isaiah Bowman 4. Ratzel

81. Which one of the following is the most suitable for representation of growth of population overtime?
1. Multiple bar diagram 3. Pie diagram
2. Line graph 4. Choropleth map

82. The term ‘Carbon Sinks’ is used for which one of the following?
1. For burning of fossil fuels 3. For forests
2. For volcanic eruption 4. For ammonifying bacteria

83. The term ‘Demanufacturing’ is associated with which one of the following?
1. Post globalization 3. Disassembly and recycling
2. Financial deficit 4. Small scale industry decline

84. Match List I with List II

LIST I LIST II
(Books) (Authors)
A. Hundred Years of Geography I. G.Taylor
B. Influences of Geographic II. T.W.Freeman
Environment
C. Geography in the Twentieth III. E.C. Semple
Century
D. Man and Nature IV. G.P. Marsh
Choose the correct answer from the options given below
1. A-II,B-III,C-I,D-IV 3. A-I,B-IV,C-III,D-II
2. A-III,B-II,C-IV,D-I 4. A-IV,B-I,C-II,D-III

85. Identify the scholar who was the first to report and map the Chinese coal fields.
1. Ritter 3. Ratzel
2. Humboldt 4. Richthofen

78
PYQ 2023 BSSEI (M-8777279548) Coaching for NET Geography
86. Largest number of tribal population (2011) in India is found in____
1. Nagaland 3. Gujarat
2. Madhya Pradesh 4. Tamil Nadu

87. Non-structural measures of flood disaster management in India include____


1. Flood zonation 3. Embankment construction
2. Flood forecasting 4. Channel improvement
Choose the most appropriate answer from the options given below
1. A and D only 3. B and C only
2. A and B only 4. B and D only

88. Which of the following scholars argued for humanistic geography?


1. Soja 3. Yi-Fu-Tuan
2. Smith 4. Schaefer

89. Which one of the following glaciers is located in Pir Panjal range of Himalayas?
1. Siachen glacier 3. Sonapani glacier
2. Biafo glacier 4. Batura glacier

90. Which among the following is the correct chronological order of Greek scholars?
A. Eratosthenes C. Hecataeus
B. Posidonius D. Hipparchus
Choose the correct answer from the options given below
1. C,A,D,B 3. D,A,B,C
2. A,D,B,C 4. B,D,A,C

91. Read the given paragraph(91-95) carefully and answer the following question.

The average rate of temperature decrease upward in the troposphere is about 6 o C per km, extending to the
tropopause. This vertical gradient of temperature is commonly referred to as normal lapse rate. Certain
processes in the lower troposphere create an actual increase in temperature with an increase in altitude, that
is, a temperature inversion. Temperature inversions may be produced in five ways (1) When the earth’s surface
loses more heat by radiation than it gains by any of the energy transfer processes. As on a clear night or at
high latitudes in winter, it cools and consequently lowers the temperature of the adjacent layer of air.
Inversions due to radiational cooling (radiation inversions) develop best in calm air under clear skies over flat
terrain. (2) Cold and dense air from hilltops and slopes tends to flow downslope to collect in valley bottoms,
creating an inverted lapse rate in the free air over the valley floor. Air drainage inversions are frequently
associated with spring frosts in middle latitudes, and for this reason fruit growers prefer gentle slopes to valley
bottoms for orchard sites. (3) When two air masses with different temperature characteristics come together,
the colder air, being more dense, tends to push underneath the warmer air and replace it. The boundary zones
along which two air masses meet are called fronts, and the inverted lapse rate which results is a frontal
inversion. Frontal inversions are not confined to the lower layers of the troposphere. They may form at upper
levels wherever cold air underruns warm air or warm air advances above cold air. (4) Advection of warm air
over a cold surface creates an inversion in the lower layers of the air mass as the warm air is cooled by
conduction. (5) The subsidence inversion forms in an air mass when a large body of air subsides and spreads
out above a lower layer. In the process the air heats dynamically more in the upper portion than at its base.
Inversions of this type may develop at considerable altitudes.

Which one of the following represents advectional inversion?

1. Movement of cold air over a warm surface


2. Movement of warm air over a cold surface
3. Movement of dense cold air from hilltops
4. Coming together of contrasting air masses

79
PYQ 2023 BSSEI (M-8777279548) Coaching for NET Geography
92. Which one of the following is NOT an ideal condition for radiation inversion?
1. Short nights 2. Clear skies 3. Flat terrain 4. Calm air

93. The preferred orchard sites in the middle latitudes are___


1. Valley bottoms 2. Ridge-side gentle slopes 3.Steep slopes 4. Fertile valley plains

94. Which one of the following is not confined to lower parts of the troposphere?
1. Radiation inversion and fog 2. Air drainage inversion and clouds
3. Frontal inversion and subsidence inversion 4. Advection inversion and frontal inversion

95. Permanent snow caps on high mountains even in the tropics is due to___
1. Drainage inversion 3. Subsidence inversion
2. Radiation inversion 4. Normal lapse rate

96. In the given diagram(96-100) regional consumption pattern of energy from various sources is
represented.
Answer the following question on the basis of the diagram.

In which of the two regions, the consumption of coal is less than hydroelectricity?
1. Europe and Africa 3. Middle East and Asia Pacific
2. Europe and North America 4. Europe and Asia Pacific

97. In which one of the following regions, Hydroelectricity is not a source of energy?
1. Africa 2. Asia Pacific 3. Middle East 4. North America

98. In which region, the combined consumption of renewables and nuclear energy is lowest?
1. South America 2. North America 3. Middle East 4. Asia Pacific

99. In which one of the region nuclear energy consumption is more than the renewable energy?
1. Asia Pacific 2. Middle East 3. Africa 4. North America

100. In which region the combined consumption of renewables and hydroelectricity is highest?
1. Africa 2. North America 3. Asia Pacific 4. Europe

80

You might also like